Archer 7

¡Supera tus tareas y exámenes ahora con Quizwiz!

Physiological needs Safety Love and belonging Esteem Self-actualization

Arrange Maslow's hierarchy of needs from the highest to the least priority.

83 gtts/minute

The primary healthcare provider (PHCP) prescribes 500 mL of 0.45% saline to be administered over one hour. The drop factor is 10 gtts/mL. The nurse sets the flow rate at how many drops per minute? Round your answer to the nearest whole number. Fill in the blank.

A Choice A is correct. Impaired gas exchange is the most appropriate nursing diagnosis for a client with iron deficiency anemia. Hemoglobin is the component in the blood responsible for transporting oxygen throughout the body. Iron is an essential substance for hemoglobin synthesis. In iron deficiency anemia, the hemoglobin is decreased, leading to impaired gas exchange. Choice B is incorrect. Iron deficiency anemia does not cause ineffective airway clearance. The nursing diagnosis of ineffective airway clearance is more likely to be caused by increased mucus secretions, an altered level of consciousness in the client, or an inadequate or absent cough reflex in the client. Choice C is incorrect. Iron deficiency anemia would not result in a fluid volume deficit nursing diagnosis. Choice D is incorrect. In relation to iron deficiency anemia, the nursing diagnosis of ineffective breathing pattern(s) is not the initial nor the most appropriate nursing diagnosis for this client's care plan. Anemia is a reduction in the number of red blood cells, the amount of hemoglobin, or the hematocrit (i.e., the percentage of packed red blood cells per deciliter of blood). Iron is needed to form hemoglobin, part of erythrocytes that carry oxygen and remove carbon dioxide (a waste product) from the body. Iron deficiency anemia is the most common type of anemia worldwide, especially among females, older adults, and individuals with poor diets. A decreased iron supply may result from blood loss, insufficient gastrointestinal absorption of iron, and/or an inadequate intake of iron via the individual's diet.

The nurse caring for a client with iron deficiency anemia is preparing the client's plan of care. Which nursing diagnosis is the most appropriate for this client's care plan? A. Impaired gas exchange B. Ineffective airway clearance C. Deficient fluid volume D. Ineffective breathing pattern

B Choice B is correct. Serotonin syndrome is a result of too much serotonin in the body due to the use of SSRI's and MAOI's. Serotonin syndrome is characterized by high body temperature, agitation, muscle rigidity, tremor, sweating, dilated pupils, and diarrhea. Upon noticing these symptoms, the nurse must report this to the physician to initiate medical intervention. Choice A is incorrect. These symptoms are not associated with serotonin syndrome. Choice C is incorrect. These symptoms are related to pulmonary edema, not serotonin syndrome. Choice D is incorrect. Weakness and diaphoresis are symptoms associated with hypoglycemia, not serotonin syndrome.

The nurse in the psychiatric unit is administering fluoxetine (Prozac) together with tranylcypromine (Parnate). The nurse should watch out for which symptoms signifying an adverse reaction from the combination of both drugs? A. Low blood pressure and urinary retention B. Muscle rigidity and hyperthermia C. Shortness of breath and pink frothy sputum D. Weakness and diaphoresis

C, D Choices C and D are correct. Since the patient is 14-years-old, they are a minor and their parents will be responsible for signing informed consent. The nurse is accountable for validating that the parents are competent to provide consent for the patient (Choice C). The nurse will serve as the witness for the informed consent. This is one of the primary responsibilities of the nurse when a patient is getting a procedure and signing a consent. The other primary responsibility will be to serve as the patient's advocate and ensure that the parents have received sufficient information to make an informed decision. If they have not, the nurse must call the surgeon to return and speak further with the parents (Choice D). Choice A is incorrect. It is not within the nurses' scope of practice to explain the procedure to the patient. It is the surgeon's responsibility to do this and they should use words/terms that the patient and family can easily understand. If the family does not speak English, a medical language interpreter should be utilized. Choice B is incorrect. It is not within the nurses' scope of practice to review the risks and benefits of the surgery with the patient. It is the surgeon's responsibility to do this. If the family does not speak English, a medical language interpreter should be used.

The nurse is caring for a 14-year-old who is scheduled to go to the OR for an appendectomy later in the day. What is the nurse's role in obtaining informed consent before surgery? Select all that apply. A. Explain the procedure to the patient in terms they can understand. B. Review the risks and benefits of the surgery. C. Validate that the parents are competent to provide consent for the patient. D. Witness the signature on the informed consent

A Choice A is correct. Bupropion is an antidepressant medication that may be used for clients with major depressive disorder (MDD). Choices B, C, and D are incorrect. Bupropion is a medication indicated for major depressive disorder. A client with bipolar mania should not receive bupropion because of the medication's activating effects. Bupropion is not an antipsychotic medication and would not decrease delusional symptoms. Finally, bupropion is not indicated for alcohol abuse disorder. Rather, it is commonly utilized for those wishing to abstain from smoking. Bupropion is a medication indicated for MDD and may be used for smoking cessation. ➢ This medication increases norepinephrine and dopamine; therefore, it has an activation effect. ➢ This medication is often advantageous because compared to the serotonergic agents (such as citalopram), this medication does not cause sexual side effects such as decreased sexual drive. ➢ This medication may worsen anxiety and is often not utilized for anxiety disorders. ➢ If the client has a history of seizure disorders, this medication would be contraindicated as it lowers the seizure threshold.

The nurse is caring for a client receiving bupropion. Which of the following findings would indicate a therapeutic response? A. A decrease in depressive symptoms B. A decrease in manic symptoms C. A decrease in delusions D. A decrease in alcohol cravings

A, D, F Choices A, D, and F are correct. Treatment for an acute migraine headache (MH) involves abortive medications such as ketorolac (NSAID), dexamethasone (corticosteroid), and acetaminophen-caffeine. Depending on the severity of the MH, the provider takes a stepwise or aggressive approach to treatment. Choices B, C, and E are incorrect. Nitroglycerin would intensify the headache and would be unhelpful during an acute MH. Topiramate is an anticonvulsant and is a preventative treatment. This medication takes time to establish efficacy and is not used for acute migraines. Opioids (fentanyl, morphine, hydromorphone) are not recommended in managing an MH because they are likely to cause a paradoxical headache. ADDITIONAL INFO The treatment for an acute migraine headache aims to abort the headache and the associative symptoms such as nausea and vomiting. Commonly, a client may be prescribed an anti-emetic such as metoclopramide to assist with abating the symptoms. The below table reviews the treatment options for a migraine headache.

The nurse is caring for a client with an acute migraine headache. The nurse would anticipate a prescription for which medication? Select all that apply. A. Ketorolac B. Nitroglycerin C. Topiramate D. Dexamethasone E. Hydromorphone F. Acetaminophen-caffeine

A Choice A is correct. Cilostazol is a phosphodiesterase inhibitor approved to treat peripheral arterial disease. Its action mechanism decreases platelet aggregation and promotes vasodilation, allowing a client to ambulate distances without pain. Choices B, C, and D are incorrect. Cilostazol is not utilized to mitigate total cholesterol levels. Further, this medication does not improve visual acuity or attention. ✓ Cilostazol is an effective treatment for a client with peripheral arterial disease (PAD). ✓ Manifestations of PAD include pain while walking (claudication), decreased peripheral pulses, and painful ulcers. ✓ Common side effects of this medication include diarrhea and headache. ✓ Under no circumstances should this medication be given to a client with heart failure as it may worsen the condition.

The nurse is caring for a client who is receiving prescribed cilostazol. Which of the following client findings would indicate a therapeutic response? A. Absence of pain while ambulating B. Decreased total cholesterol C. Increased visual acuity D. Improved focus and attention

A, B, D, E Choices A, B, D, and E are correct. A port is a central venous line that is useful for individuals receiving chemotherapy. The nurse should utilize an aseptic technique to prevent central line-associated bloodstream infections (CLABSIs) when the port is accessed. This includes the nurse and the client wearing a mask as well as the nurse using sterile gloves. Occlusion is a common complication with a port, and prior to de-accessing, the nurse should flush heparin. Further, the client should be instructed to wear a mask to prevent contamination during dressing changes. Finally, the nurse must verify appropriate access by aspirating for blood return prior to medication administration. Choice C is incorrect. The nurse utilizes a non-coring needle to access a port. A 16-gauge catheter will be an option if a nurse starts a large-bore peripheral IV. When a port is accessed, it is accessed with a non-coring needle that is 0.5 to 2 inches, with the gauge being 19 to 22. When caring for a client with a port, it is essential to prevent CLABSIs through meticulous hand hygiene and aseptic dressing changes. The nurse should instruct the patient to avoid getting the accessed port wet and report signs of infection such as erythema. If the port is not in use, it needs to be accessed and flushed once a month to maintain patency.

The nurse is caring for a client with a port. Which of the following actions would be appropriate to take? Select all that apply. A. Access the port using sterile technique. B. Flush the port with heparin prior to de-access. C. Access the port using a 16-gauge catheter. D. Have the client wear a mask during the dressing change. E. Aspirate for blood return prior to medication administration.

A, B, C, E Choices A, B, C, and E are correct. These statements are true and should be included in the conference. Mittens are not considered restraints if they are untethered, and the client may be able to remove the mitt. Elbow restraints make removing a medical device near the face or neck difficult. It does not impede the removal of abdominal or urinary medical devices. Belt restraints should be applied over a client's clothing and secured to the bedframe. Significant impairment to a client's skin may result if the belt is directly applied over the skin. Restraints must be removed every two hours to allow for a range of motion. However, if the client is violent, each restraint should be removed once at a time versus both, which allows the client to elope or inflict further violence. Choice D is incorrect. No matter the type, physical restraints cannot be utilized on an as-needed (PRN) basis. Physical restraint usage should be appropriately justified based on the client's behavior during the assessment. This information should not be included because it is incorrect. ✓ Restraints should be used as a last resort if alternative methods are ineffective. ✓ A nurse should never threaten a client with restraints. This is considered assault. ✓ The nurse may place a client who is violent in restraints without an order from the primary healthcare provider (PHCP). If this were to occur, the nurse would have one hour to inform the provider and obtain an order. ✓ Restraints are never as needed (PRN). They should be discontinued at the earliest possible time. ✓ When restraining a client, the reason for the restraint must be explained to the client and the behavior the client needs to demonstrate for the restraints to be discontinued. ✓ The nurse should observe the client at frequent intervals to offer nutrition & toileting, assess their behavioral status, obtain vital signs, and provide range of motion. These intervals are determined by the facility and the type of restraint—the more restrictive the restraint and the younger the client, the more frequent assessment. ✓ Restraints must be able to quickly be removed via a quick-release buckle (knots are no longer recommended). ✓ The nurses' documentation must be comprehensive, describing the reasoning for the restraints, alternatives utilized, the education provided to the client, the type of restraint utilized, how it was secured, and the ongoing behavior necessary to continue the restraint. The nurse should also document the intervals at which the restraints were released.

The nurse is planning a staff development conference about restraints. Which of the following information should the nurse include? Select all that apply. A. Mittens are not restraints if untethered and the client is physically able to remove the mitt. B. Elbow restraints may allow a client to remove abdominal or urinary medical devices. C. Soft wrist restraints should be removed one at a time if a client is violent. D. Belt restraints may be prescribed on an as-needed basis. E. Belt restraints should be applied over a client's clothing garments.

C Choice C is correct. The nurse should apply a tourniquet above the client's residual limb to stop the bleeding. This should be the nurse's first intervention. Choice A is incorrect. The nurse may increase the client's IV but not after implementing measures that can stop the bleeding. Choice B is incorrect. The nurse should assess the client's vital signs but not until after stopping the bleeding. Choice D is incorrect. The nurse needs to notify the physician but only after stopping the bleeding.

The nurse notices some bright red blood on the residual limb dressing of a client that had a below-the-knee amputation. The nurse suspects an arterial bleed. What should be the nurse's first action? A. Increase the IV rate. B. Take the client's vital signs. C. Apply a tourniquet above the amputation. D. Notify the physician.

A Choice A is correct. This patient is presenting with early signs/symptoms consistent with compartment syndrome. Later signs of compartment syndrome include paralysis and the absence of pulses in the affected extremity. If not caught and treated early, compartment syndrome can result in permanent muscle and nerve damage. Choice B is incorrect. There is no assessment data that supports a diagnosis of sepsis. Localized pain is expected in fracture patients, but would not be indicative of sepsis without additional symptoms of infection. Choice C is incorrect. Patients with nerve damage/peripheral neuropathy would experience symptoms such as pain and tingling. Still, the other assessment data of cold skin temperature and weak pulse would not support this diagnosis. Peripheral neuropathy would not be a more significant concern than compartment syndrome. Choice D is incorrect. This patient would be at risk of developing a pressure injury due to injury and immobility, but this potential complication would not be a more significant concern than compartment syndrome.

The patient with a right distal fibula fracture complains of pain and a tingling sensation in the right foot. Upon assessment, the nurse notes the right foot is cold to the touch with a weak dorsalis pedis pulse. Which potential complication should the nurse be most concerned about? A. Compartment syndrome B. Sepsis C. Peripheral neuropathy D. Pressure Injury

C Choice C is correct. First, determine if the ABG is compensated or uncompensated. Because the pH is between 7.35 and 7.45, it is compensated. You know this because the pH is normal, but the CO2 and HCO3 are not. Next, determine if it is acidosis or alkalosis. The pH is closer to 7.35, which anything less than would be acidotic, so it is an acidosis. Lastly, determine if it is respiratory or metabolic. To do this match, which value, CO2 or HCO3, coincides with the pH. CO2 is acidic, and HCO3 is basic. In this example, we have an acidosis, so the CO2 is what corresponds, making this a respiratory issue. Putting it all together, this case would be compensated respiratory acidosis. This patient is retaining CO2, which is causing them to become acidotic. In response, the kidneys are increasing the production of bicarbonate to bring the pH back into a healthy range. They have been able to compensate for respiratory acidosis. Choices A, B, and D are incorrect.

Analyze the following ABG: pH 7.36, CO2 69, HCO3 37 A. Compensated metabolic acidosis B. Uncompensated metabolic acidosis C. Compensated respiratory acidosis D. Uncompensated respiratory alkalosis

B Choice B is correct. The nurse should observe the client while eating and prevent the client from using the restroom for 90 minutes each meal to break the purging cycle. Although purging is traditionally associated with bulimia, purging is actually seen in both in both anorexia and bulimia (while this sounds very similar to bulimia, the primary difference is that in bulimia, clients are of normal or above-normal weight, whereas those with anorexia nervosa (even the binge eating/purging type) have a significantly low body weight). Binge eating with purging occurs in 30 to 50% of anorexia clients; therefore, the post-meal restroom restriction is needed to ensure anorexia clients are not purging. Choice A is incorrect. Beyond a very low intensity exercise (i.e., walking at a slow pace, stretching) for a short period of time, exercise should not be encouraged until the client has shown adequate weight gain and met the eating disorder treatment program's requirements before allowing the client to engage in exercise. Choice C is incorrect. There is no specific indication for the client to lie down after meals. Choice D is incorrect. Encouraging an intense exercise program would be detrimental to the client and is therefore contraindicated. The maximum amount of exercise the client should be allowed to participate in would be a very low intensity workout (i.e., walking at a slow pace, stretching, etc.) for a short duration of time. Exercise should not be encouraged until the client has shown adequate weight gain and met the eating disorder treatment program's requirements before allowing the client to engage in exercise. Anorexia nervosa occurs predominantly in girls and young women. The onset of this disease usually occurs during adolescence and rarely occurs after age forty. Two types of anorexia nervosa are recognized: In the restricting type of anorexia nervosa, clients restrict food intake but do not regularly engage in binge eating or purging behavior. Additionally, some of these clients exercise excessively. In those with the binge eating/purging type of anorexia nervosa, clients regularly binge eat before inducing vomiting and/or misusing laxatives, diuretics, or enemas. Unlike bulimia clients (who are of normal or above-normal weight), these clients have a significantly low body weight. Clients with anorexia nervosa have an intense fear of gaining weight or becoming fat that persists despite all evidence to the contrary. In adults, BMI is significantly low (usually BMI of < 17 kg/m2), and in adolescents, BMI percentile is low (usually < 5th percentile) or does not increase as expected for normal growth. Endocrine or electrolyte abnormalities or cardiac arrhythmias may develop, potentially resulting in death. Treatment consists of nutritional supplementation and psychotherapy (e.g., cognitive behavioral therapy). For adolescents, family-based therapy is often utilized.

At an eating disorder treatment center, a nurse is caring for a client with anorexia nervosa who has recently arrived at the facility. Which intervention should the nurse apply following the client's meals? A. Instruct the client to exercise by going for a walk following meals B. Restrict the client from using the restroom for 90 minutes after each meal C. Ask the client to lie down for two hours after each meal D. Encourage the client to begin an intense exercise program, with short exercise sessions after each meal

C Choice C is correct. Ergonomically designed chairs are commonly designed with a primary focus on providing lumbar spine support. Although the chairs often provide some level of support to various levels of the spinal column, the lumbar spine is the most common region for back pain to occur and therefore is the spinal region ergonomically designed chairs routinely support. Choice A is incorrect. The cervical spine is not typically supported by ergonomically designed chairs. Choice B is incorrect. Although ergonomically designed chairs do support the thoracic spine when an individual leans back, proper positioning would often have an individual sitting upright at a 90-degree angle. Choice D is incorrect. While ergonomically designed chairs do support the sacral spine, the chairs concentrate on the lumbar region of the back primarily due to society's high prevalence of lumbar back pain complaints. The lumbar spine—where most back pain occurs—includes five vertebrae (L1-L5). The lumbar region supports the majority of the upper body weight.

Ergonomically designed chairs are best designed to provide support to which region of the spine? A. The cervical spine B. The thoracic spine C. The lumbar spine D. The sacral spine

A Choice A is correct. Dysphagia indicates an inability to swallow. Parenteral nutrition aims to meet the body's need for nutrients via a route other than the GI tract (i.e. bloodstream) and is indicated when the GI tract is unable to ingest, digest, or absorb nutrients. A patient who is unable to swallow may require an alternate form of nutrient delivery, such as enteral nutrition, but this would not necessarily be an indication for parenteral nutrition. Choice B is incorrect. A gastrointestinal obstruction would interfere with the GI tract's ability to ingest, digest, and absorb nutrients, so this is a common indication for parenteral nutrition. Choice C is incorrect. Severe anorexia nervosa would interfere with the GI tract's ability to ingest, digest, and absorb nutrients, so this is a common indication for parenteral nutrition. Choice D is incorrect. Severe burns or trauma can put a greater demand on the body for nutrients than what can safely be consumed via the GI system. Parenteral nutrition may be indicated to supplement the additional nutrients without overloading the GI system with excessive caloric volume.

The RN is caring for a 72-year-old patient on the medical-surgical floor. Which of the following factors would not be an indication for this patient to receive parenteral nutrition? A. Dysphagia B. Gastrointestinal obstruction C. Severe anorexia nervosa D. Severe burns

A, B, D, E Choices A, B, D, and E are correct. This client data reflects that the client is ready for discharge home. The client has a positive gag reflex, adequate urinary output (UOP) for the postoperative time frame (> 30 mL/hr), positive bowel sounds, and minimal pain. The client's UOP is high, but it is only would be concerning if it were low. Intraoperative IV fluids may be given to explain the surgery that explains the increased UOP. Hypoactive bowel sounds immediately following anesthesia is expected because anesthesia decreases peristalsis. Absent bowel sounds would be a concerning finding. The client's pain is minimal and does not inhibit their ability to be discharged. Choice C is incorrect. This blood pressure is clinical hypotension and requires correction before discharge. It would be unsafe to discharge the client with this low blood pressure. The nurse should report this finding to the primary healthcare provider. For a client to be discharged following a surgery, the following criteria must be met: ➢ Return to baseline neurological status ➢ Adequate urinary output (30 mL/hr) ➢ Return of reflexes (cough, gag, swallow) ➢ Bowel sounds present in all four quadrants ➢ Ability to ambulate ➢ Vital signs within normal limits ➢ Ability to tolerate oral fluids ➢ Minimal nausea and vomiting ➢ Adequate pain control

The nurse is caring for a client three hours postoperative following a laparoscopic appendectomy. Which of the following client data indicates the client is ready for discharge home? Select all that apply. A. Positive gag reflex B. Hypoactive bowel sounds C. Blood pressure 90/60 mm Hg D. Incisional pain '2' on a scale of 0 to 10 E. Urinary output of 240 mL since surgery

C Choice C is correct. Patients who have undergone a femoral vein catheter should refrain from sitting up more than 45 degrees because this could kink the catheter, thus interfering with treatment. Choice A is incorrect. This patient does not need to be on a fluid restriction unless specifically indicated by the physician. Choice B is incorrect. Toe touches require that the patient bend more than 45 degrees and may damage or kink the femoral vein catheter. Choice D is incorrect. The patient should leave their occlusive dressing on while the femoral vein catheter is inserted.

The nurse is caring for a patient who has recently had a femoral vein catheter placed. The nurse would be most correct in advising the patient to do which of the following? A. Refrain from drinking more than 500 mL per day B. Perform toe touch stretches in bed every morning C. Refrain from sitting up more than 45 degrees D. Remove the dressing if it becomes itchy

B Choice B is correct. Weight gain is an expected finding in pregnancy and is not a symptom that requires investigation into possible abuse. A woman's risk of becoming a victim of domestic violence increases when pregnant. The health care team should be diligent in watching for signs of violence and abuse in the prenatal client. Choices A, C, and D are incorrect. Depression, unexplained bruising, and late initiation of prenatal care are all signs of possible abuse. Other symptoms include drug or alcohol abuse, chronic pain, and isolation.

The nurse is working with a woman who is five months pregnant and attending her first prenatal appointment after completing the client's history. The nurse suspects that she is a victim of domestic violence. Which of the following is not a sign of domestic violence? A. Depression B. Weight gain C. Unexplained bruising D. Late initiation of prenatal care

Inspect the bottle for the type of insulin and the expiration date. Gently roll the bottle of intermediate-acting insulin in the palms of your hands to mix the insulin. Clean the rubber stopper with an alcohol swab. Pull back the plunger to draw air into the syringe and inject it into the vial. Turn the bottle upside down and draw the insulin dose into the syringe. Remove air bubbles in the syringe by tapping on the syringe.

The nurse prepares to administer intermediate-acting insulin to a client with diabetes mellitus. Arrange the steps in the order that the nurse must perform them when preparing the injection. All options must be used.

12 units

The primary healthcare provider (PHCP) prescribes a bolus of regular insulin prior to a continuous infusion. The prescription is for 0.1 units/kg. The client weighs 256 lbs. How many units of insulin should the nurse administer to the client? Fill in the blank. Round your answer to the nearest whole number.

B, C Choices B and C are correct. Computerized records have boxes to check and choices to make so that nurses do not have to write assessment findings by hand each time they evaluate a patient. They also have room for adding free text (Choice B). Computerized Provider Order Entry (CPOE) allows providers to enter all orders directly into the computer, electronically communicating requests to the pharmacy, laboratory, and nursing unit (Choice C). Choice A is incorrect. Although there is no hard copy of the medical record, the computerized medical history is still considered a legal document and can be used in a lawsuit. Choice D is incorrect. Implementing a computerized record system is expensive, and it requires much planning and education. However, it does significantly increase patient safety concerns.

Which of the following are true regarding an electronic medical record? Select all that apply. A. It cannot be used as a legal document in a lawsuit. B. Nurses can enter data by checking boxes and adding free full text. C. It allows primary care providers to directly order from the computer. D. It is economical as well as easy to learn and implement

C Choice C is correct. The cricoid appears as a full circular ring and is the most narrow part of the airway. While intubating, it can be useful to place pressure on the cricoid to make the airway more comfortable to access. Choice A is incorrect. The hyoid is a semi-circle ring, not a circular ring. It helps support the tongue. Choice B is incorrect. The arytenoid muscle is at the back of the larynx and allows the vocal cords to work correctly. Choice D is incorrect. The thyroid is an organ that sits below the "Adam's apple" and is not a part of the airway.

Which part of the laryngeal cartilage is a full circular ring and is the narrowest part of the airway in young children? A. Hyoid B. Arytenoid C. Cricoid D. Thyroid

A Choice A is correct. Vomiting (emesis) is initiated by a nucleus of cells located in the medulla called the vomiting center. This center coordinates a complex series of events involving pharyngeal, gastrointestinal, and abdominal wall contractions that lead to the expulsion of gastric contents. Choice B is incorrect. Catecholamine inhibition does not induce vomiting. Choice C is incorrect. Chemotherapy does not induce vomiting from autonomic instability. Choice D is incorrect. Chemotherapy, especially oral agents, may have an irritating effect on the gastric mucosa, which could result in afferent messages to the solitary tract nucleus. Still, these pathways do not project to the vomiting center.

Chemotherapy induces vomiting by: A. Stimulating neuroreceptors in the medulla. B. Inhibiting the release of catecholamines. C. Autonomic instability. D. Irritating the gastric mucosa.

A Choice A is correct. The initial action should be to assess the client's home for safety modifications. Grab bars in the bathroom give the client leverage when getting up. Choice B is incorrect. Taking a shower may be safer, but the nurse should first assess the client to find out if safety equipment is present in the home. Choice C is incorrect. Referral to a physical therapist could be done for the patient to improve strength and mobility; however, the initial priority must be client safety. Choice D is incorrect. NSAIDs aid the client in mobility by decreasing the pain felt. However, safety must first be ensured by the nurse.

The client with osteoarthritis tells the nurse, " I have problems getting up from the toilet bowl." What would be the next action by the nurse? A. Check if the client has grab bars in the bathroom. B. Instruct the client to take a shower. C. Refer the client to a physical therapist. D. Ask if the client takes nonsteroidal anti-inflammatory drugs (NSAIDs).

A, E Choices A and E are correct. Cystic Fibrosis is a multisystem disorder that causes gastrointestinal disturbances such as malabsorption of essential fat-soluble vitamins (A, D, E, and K). A multivitamin is prescribed to help mitigate these vitamin deficiencies. Salmeterol is a long-acting bronchodilator and has utility in cystic fibrosis as the airways may become narrowed or obstructed. Choices B, C, and D are incorrect. These medications are not utilized in cystic fibrosis. Cystic fibrosis is a multisystem disorder that has no cure. A well-balanced diet rich in calories, protein, and fat is recommended to help prevent (or treat) the malabsorption associated with CF. Foods rich in sodium are also recommended because of the salt loss through the skin. A multivitamin is commonly prescribed to help mitigate the vitamin deficiencies that may develop.

The nurse is reviewing a client's list of medications who has cystic fibrosis. The nurse anticipates a prescription for which medication? Select all that apply. A. Multivitamin B. Aspirin C. Warfarin D. Simvastatin E. Salmeterol

B, D Choices B and D are correct. To provide adequate nutrition and gas exchange for the developing fetus, a woman's body undergoes several changes during pregnancy, including cardiovascular, hematologic, metabolic, renal, and respiratory changes. In a healthy pregnancy, maternal blood volume may increase by as much as 40 to 50% by week 32 of the pregnancy. Despite this increase in red blood cell production, the mother may develop dilutional physiological anemia. Mild to moderate fatigue may be experienced. During the second trimester, the nurse might note a decrease in diastolic blood pressure. Cardiac output may decrease as the mother changes positions. Choice A is incorrect. The thyroid and pituitary glands typically increase in size during pregnancy, not decrease. Reflecting the increased metabolic needs during pregnancy, TSH (thyroid-stimulating hormone) increases, and therefore, thyroid volume increases. Choice C is incorrect. Intestinal mobility decreases as progesterone levels increase to allow for increased absorption of nutrients. The nurse should be aware that this change may also increase the risk of constipation.

A 25-year-old female client at ten weeks gestation has mild fatigue. While reassuring her that this is expected, the nurse also knows that which of the following are regular changes during various trimesters of a healthy pregnancy? Select all that apply. A. Thyroid gland decreases in size B. Maternal blood volume increases C. Intestinal mobility increases D. Diastolic blood pressure decreases

A, B, C, D, F Choices A, B, C, D, and F are correct. It is essential to offer the client an opportunity to express his/her feelings. The best way to do this is to simply ask how they are feeling or what they think about a particular situation (Choice A). Providing foods that are compatible with the client's religious beliefs adds to the feeling of self. In some cases, it may be acceptable to encourage family members to bring food from home (Choice B). Help the patient identify feelings of guilt. You might ask the following after a patient has voiced a concern: "How do you feel about that?" or "You seem to feel sad about saying/doing that" (Choice C). Maximize the patient's comfort. This is one of the most important spiritual activities a nurse can perform. A patient cannot think about religious issues when plagued with physical pain or discomfort (Choice D). Assess the patient's needs for reconciliation. This may include reconciliation with self, others, and God (Choice F). Choice E is incorrect. The nurse should not make assumptions about the patient's and family's beliefs.

A client is experiencing spiritual distress after receiving a diagnosis of advanced brain cancer. Which of the following would be appropriate interventions? Select all that apply. A. Ask the patient how he/she is feeling. B. Provide food compatible with the person's religious needs. C. Help the patient identify feelings of guilt. D. Offer to massage the client's shoulders. E. Offer encouragement based on assumptions about the patient's beliefs. F. Assess the patient's needs for reconciliation.

C Choice C is correct. Flushing of the face is a response to the intravenous administration of contrast dye commonly seen in clients. Many diagnostic and imaging procedures (CT scans, angiograms, myelograms, etc.) involve intravenous contrast (intravenous dye, iodinated contrast) dyes containing iodine. Often, clients experience a warm sensation throughout the body once the intravenous contrast dye begins infusing. Usually, if the client does experience this warm sensation, clients will typically report the sensation occurring initially in the face and neck region. Shortly thereafter, clients will often state they feel the warmth in their pelvic region. Choice A is incorrect. Chest tightness is not an anticipated response following the administration of an intravenous contrast dye. The client's report of chest tightness may likely indicate the client is experiencing a moderate to severe hypersensitivity reaction to the intravenous contrast dye. At the first sign of a reaction, the contrast agent is stopped. Choice B is incorrect. Burning at the intravenous infusion site is not an anticipated outcome or side effect when administering intravenous contrast dye. Therefore, this statement by the nurse should not be included in the client's nursing education. Choice D is incorrect. Contrast-induced acute kidney injury (CI-AKI) is a rare, but severe, adverse effect of intravenous (iodinated) contrast media, resulting in decreased (or ceasing of) kidney functioning. The nurse educating the client that an allergic reaction may cause a decline in the client's kidney function is incorrect and would be a misrepresentation. In order to reduce the risk of CI-AKI, all clients scheduled to receive intravenous contrast dye must have an eGFR (estimated glomerular filtration rate) available prior to the test (typically within 48 hours of the test or per facility policy). Intravenous contrast dye must not be given if the estimated glomerular filtration rate (eGFR) is less than 30 mL/min/1.73 m2. For those at risk of renal toxicity, intravenous hydration must be given following contrast-containing procedures. (Of note, although not indicated in the above scenario, renal function testing may be bypassed if the medical acuity of the client warrants immediate contrast-enhanced imaging (e.g., potential acute cerebrovascular accident, potential acute aortic dissection, Level 1 trauma, etc.). Allergic reactions to intravenous contrast dyes occur in only 5-8% of clients. The contrast media acts directly to release histamine and other mediators from mast cells. No allergic antibody mediates this reaction; hence, it is referred to as a "pseudo-allergy."

A client is scheduled for a CT angiogram, which involves the administration of an intravenous contrast dye. In preparing the client for the procedure, the nurse's client education should include which of the following statements? A. "You should expect some chest tightness during the procedure." B. "You should expect a burning sensation at the intravenous site." C. "You may experience flushing of the face." D. "An allergic reaction may cause a decline in your kidney function."

C Choice C is correct. Vitamin C increases the absorption of iron. Due to the vitamin C concentration in orange juice, consuming the iron supplement with orange juice would aid in the absorption of the iron supplement. Choice A is incorrect. Products containing calcium decrease the absorption of iron in the gastrointestinal tract. Therefore, avoid taking iron supplements with milk. More specifically, if possible, avoid the intake of milk, calcium, calcium supplements, and/or any antacids two hours before or after taking an iron supplement. Choice B is incorrect. Many of the nutrients contained within a meal will hinder, at minimum, a portion of the iron supplement's absorption within the gastrointestinal tract. Choice D is incorrect. Iron supplements often cause constipation in clients. If this occurs, the client may find it necessary to increase water intake. Iron is responsible for oxygen distribution to hemoglobin and myoglobin. Iron is absorbed the best on an empty stomach. However, iron supplements can cause stomach cramps, nausea, and diarrhea in some individuals. Some clients may need to take iron with a small amount of food to circumvent these problems.

A client visits an antepartum clinic for her check-up and is prescribed iron supplements. Which instruction should the nurse provide to the client regarding her iron supplement therapy? A. You must take the iron supplements with milk. B. Consume the iron supplements with meals. C. Take the iron supplement with orange juice. D. Avoid consuming large amounts of water when taking the iron supplement, as this can cause nausea.

C Choice C is correct. The client has been prescribed lactulose for portal-systemic encephalopathy (PSE), not for constipation. Lactulose promotes ammonia excretion in the stool by cleansing the bowels and ridding the intestinal tract of the toxins that contribute to encephalopathy. Lactulose increases osmotic pressure to draw fluid into the colon and prevents the absorption of ammonia in the colon. The drug's desired effect is the production of two or three soft stools per day and a decrease in the client's confusion caused by increased ammonia. When observing for a response to lactulose, the client may report intestinal bloating and cramping. Serum ammonia levels may be monitored but do not always correlate with symptoms. Choice A is incorrect. The prescribed dose of lactulose is required for the client's portal-systemic encephalopathy. Withholding this medication could have severe repercussions. Choice B is incorrect. The prescribed dose is 20 grams, which is equivalent to 30 mL. Administering 3 mL instead of 30 mL would be a medication error. Choice D is incorrect. Instructing this client to take additional laxatives while on lactulose would be inappropriate, as the client has already verbalized a correct understanding of the need to avoid other laxatives. Since lactulose is used to promote the excretion of ammonia in stool, the dose must be carefully adjusted, so the client averages two to three loose bowel movements per day. Determining the optimal daily dose of lactulose becomes challenging if other laxatives are used in conjunction. Due to unfavorable taste, lactulose is often mixed with fruit juice, water, or milk to improve flavor. Hypokalemia and dehydration may result from excessive stools. Remind unlicensed assistive personnel (UAP) to assist the client with skincare (if needed) to prevent breakdown caused by excessive stools.

A client with portal-systemic encephalopathy is prescribed lactulose 20 grams orally QID. The medication is available in 3.33 grams per 5 mL oral solution. The nurse is preparing to administer a 30 mL dose of lactulose to this client. When the nurse approaches the client, the client states, "I understand that I cannot take other laxatives with lactulose." Which of the following actions should the nurse perform next? A. Withhold the lactulose B. Give only 3 mL of lactulose instead of 30 mL C. Give 30 mL of lactulose with juice and monitor blood ammonia D. Correct the client's statement by stating additional laxatives may be taken

B Choice B is correct. Chronic renal failure causes decreased testosterone levels. Low testosterone results in reduced sex drive. Sexuality has physical and emotional components, both of which can be affected by chronic kidney disease. Kidney disease can cause chemical changes in the body, affecting circulation, nerve function, hormones, and energy levels. Also, any underlying health conditions that contribute to CKD, like hypertension or diabetes, can affect male sexuality. Fatigue is one of the most common symptoms of men with kidney disease. Since kidney disease affects the endocrine system, changes in hormone levels may result in decreased sex drive. An estimated 20 to 30 million men in the U.S. have problems with impotence. Erectile Dysfunction can happen when blood vessels and nerves to the penis become damaged. Without proper blood flow, the penis cannot maintain an erection. Diabetes and high blood pressure affect blood flow and weaken blood vessels. Feelings of sexual attraction become sparse when the body undergoes these unexpected changes. This can affect how people interact with others and their ability to develop intimate relationships. Men may feel worried, anxious, and depressed when faced with CKD. This is normal, but these emotions may cause loss of energy and lower interest in activities, including sex. Choice A is incorrect. Retrograde ejaculation is not a complication of chronic renal failure. It is a complication of transurethral resection of the prostate. Choice C is incorrect. In chronic renal failure, the testicles atrophy rather than hypertrophy. Choice D is incorrect. Chronic renal failure often produces a state of depression, not euphoria.

A male client with chronic renal failure has questions regarding the effects of his kidney disease on his sexual activity. Which of the following is a sexual complication of chronic renal failure? A. Retrograde ejaculation B. Decreased testosterone C. Hypertrophy of the testicles D. Feelings of euphoria

D Choice D is correct. Reinforcement of client teaching is needed. The nurse must re-educate the client on the importance of completing the entire course of this medication regardless of the perceived symptom improvement to ensure the bacterial organism is eradicated, thereby decreasing the likelihood that bacteria will develop resistance and not be treatable by antimicrobials. Choice A is incorrect. There is no need to hold the administration of this medication to clarify with the HCP, as this medication and dosage are both indicated and safe. Choice B is incorrect. Administering one tablet would be a medication error. Choice C is incorrect. No monitoring is required unless a client receives long-term antibiotic therapy with this medication. In the event long-term use was needed, renal, hepatic, and hematopoietic (i.e., not cardiac) studies would be intermittently assessed. Assess for allergies to penicillins or cephalosporins, as this medication would be contraindicated in these clients. Each dose should be taken with a meal or snack to reduce the possibility of gastrointestinal upset.

A nurse is assigned to care for a client with pneumonia. The health care provider (HCP) has prescribed "amoxicillin/clavulanate potassium 500 mg PO TID" for treatment. The medication is available in 250 mg tablets. After calculating the dosing, the nurse finds that the client will receive two tablets every 8 hours. When verifying the client's understanding, the client states, "I should take the medication with food, and if I feel fine before I finish the prescription, I can stop it and save the rest of the pills for the next time I get pneumonia." After rechecking the dosage calculation, the nurse decides to do which of the following? A. Hold medication administration and clarify the dosage with the HCP B. Administer one tablet only C. Administer the medication as prescribed and monitor the client's cardiac function D. Reinforce client teaching and administer the medication as prescribed

Verify and confirm that the code strip corresponds to the meter code. Disinfect the client's finger with an alcohol swab. Prick the side of the finger using the lancet. Turn the finger down so the blood will drop with gravity. Wipe off the first drop of blood using sterile gauze. Collect the next drop on the test strip. Hold the gauze on the client's finger after the specimen has been obtained. Read the client's blood glucose level on the monitor.

A nurse is in-training for the correct way to monitor blood glucose levels. Arrange the following steps of the blood glucose monitoring technique in the correct sequence.

B Choice B is correct. Phenytoin is an anticonvulsant and is indicated for epilepsy. Therapeutic levels must be maintained to ensure the effectiveness of the drug. The therapeutic drug levels of phenytoin are 10-20 mcg/mL. Choices A, C, and D are incorrect. Phenytoin is an anticonvulsant medication that requires adherence to prevent seizure activity. The client should not stop the drug because of the side-effect of gingival hyperplasia; instead, the client should report this effect. The client's self-discontinuing the medication increases the risk of a seizure. The client should not increase the drug if they have a seizure. Phenytoin can be taken with or without food. This medication does not have any dietary restrictions.

A nurse is instructing a client about a newly prescribed medication, phenytoin. Which statements, if made by the client, indicate effective teaching? A. "If my gums get irritated and large, I can stop this medication." B. "I will need laboratory work to monitor the medication level." C. "It is okay for me to increase this medication if I have a seizure." D. "I should take this medication with low protein foods."

B Choice B is correct. A thoracentesis is best performed with the client sitting upright and leaning slightly forward with arms supported. Unless there is a large volume of fluid in the pleural space, thoracentesis usually takes 10 to 15 minutes. During this time, most clients sit quietly on the edge of a chair or bed with their head and arms resting on a pillow positioned on a bedside table. Semi-Fowler's position is not utilized. Choice A is incorrect. The nurse should verify that the client has signed the informed consent form. If the client has not or the client has questions regarding the thoracentesis before signing the informed consent, the nurse should arrange for the physician to come to speak to the client. Choice C is incorrect. Typically, a local anesthetic (such as 1% lidocaine) is utilized to numb the area. Although numb, the insertion of the thoracentesis needle may be painful. Any sudden movement by the client could potentially force the hand through the pleural space, resulting in injuries to the visceral pleura and/or lung parenchyma. Therefore, the client should be instructed to remain still throughout the procedure. Choice D is incorrect. Any client undergoing a thoracic procedure should be monitored for tachypnea, dyspnea, and cyanosis (at minimum). Although complications are an overall rare occurrence in thoracentesis clients, when complications do occur, one of the more common complications is a pneumothorax. Informed consent is part of the health care provider-patient relationship. Informed consent must be obtained and witnessed when the client is not under the influence of medication such as opioids. The person responsible for performing the procedure is responsible for obtaining informed consent. It has been standard practice at many facilities to obtain a chest x-ray after thoracentesis to rule out pneumothorax, document the extent of fluid removal, and view lung fields previously obscured by fluid.

A nurse is preparing a client for a thoracentesis. All of the following are appropriate actions by the nurse except: A. Ensure the client has provided informed consent. B. Place the client in semi-Fowler's position. C. Instruct the client to remain still when the needle is inserted. D. Monitor for tachypnea, dyspnea, and cyanosis.

A Choice A is correct. It is appropriate for the RN to delegate to the LPN/VN to collect data on the client's neurovascular status (pulse, skin condition, capillary refill) every two hours while the client is restrained. Choices B, C, and D are incorrect. The RN is responsible for educating the client on the use of restraints. Additionally, the initiation of peripheral vascular access (intravenous line) is within the scope of the RN. Assessing the client to determine the continued need for the restraint will be necessary and is appropriate for the RN to perform. Restraints should be used as a last resort if alternative methods are ineffective. A nurse should never threaten a client with restraints. This is considered assault. The nurse may place a violent client in restraints without an order from the primary healthcare provider (PHCP). If this occurs, the nurse has one hour to inform the provider and obtain an order. Restraints are never as needed (PRN). They should be discontinued at the earliest possible time. When restraining a client, the reason for the restraint must be explained to the client and the behavior the client needs to demonstrate for the restraints to be discontinued. The nurse should observe the client at frequent intervals to offer nutrition & toileting, assess their behavioral status, obtain vital signs, and provide a range of motion. These intervals are determined by the facility and the type of restraint—the more restrictive the restraint and the younger the client, the more frequent assessment. Restraints must be able to quickly be removed via a quick-release buckle (knots are no longer recommended). The nurses' documentation must be comprehensive, describing the reasoning for the restraints, alternatives utilized, the education provided to the client, the type of restraint utilized, how it was secured, and the ongoing behavior necessary to continue the restraint. The nurse should also document the intervals at which the restraints were released.

A registered nurse (RN) and a licensed practical/vocational nurse (LPN/VN) are caring for a client who is violent and self-discontinued their peripheral vascular access. After initiating physical wrist restraints, which of the following tasks may the RN delegate to the LPN? A. Collect data on the client's skin integrity. B. Educate the client on the need for restraints. C. Initiate peripheral vascular access. D. Continually assess the client to determine if restraint use is necessary.

D Choice D is correct. An advantage of mutual pretense at the end of life for the client is that it allows the client to preserve a degree of dignity and privacy at the end of life. Mutual pretense is one of the three levels of awareness that occur with a terminal illness and at the end of life. It occurs when the dying client and the loved ones simply do not talk about death with each other even though all are aware of the seriousness of the illness and the fact that the end is near. Choice A is incorrect. Denial is not an advantage of mutual pretense. Denial is the subconscious inability of the client to believe that they are genuinely terminally ill and dying; still, they simply do not talk about it. Choice B is incorrect. The mutual pretense is not used by the client to exercise control over their loved ones at the end of life; in fact, mutual deceit is often used to protect loved ones from sorrow and distress. Choice C is incorrect. Projection is not an advantage of mutual pretense. Screening is defined as redirecting anger toward a more socially acceptable target; mutual deceit is often used to protect loved ones from sorrow and distress, not the redirection of rage.

An advantage of mutual pretense at the end of life for the client is that it allows the client: A. To fully employ the ego defense mechanism of denial at the end of life. B. To exercise control over loved ones when they are at the end of life. C. To fully employ the ego defense mechanism of projection at the end of life. D. To preserve a degree of dignity and privacy at the end of life.

B Choice B is correct. An idiosyncratic reaction is an abnormal and unexpected response to a medication, other than an allergic reaction, that is peculiar to an individual client. This type of reaction is nearly always unpredictable. Although a wide variety of idiosyncratic (i.e., unpredictable) drug reactions may occur, some examples include Stevens-Johnson syndrome or exfoliative dermatitis. Based on the answer choices, this is most likely what has occurred in this client. Therefore, Choice B is correct. Choice A is incorrect. An allergic reaction is a type of adverse drug event where an immunologic hypersensitivity reaction results from the unusual sensitivity of a patient to a particular medication. In this type of adverse event, physical manifestations of this response range from skin erythema or mild rash to severe, even life-threatening reactions such as the constriction of bronchial airways and tachycardia. Based on the information provided, this client does not seem to be experiencing an allergic reaction. Therefore, Choice A is incorrect. Choice C is incorrect. The term "adverse effect" is an umbrella term used to describe undesirable effects that are a direct response to one or more medications taken by a client. While an adverse effect did occur, more than one of the answer choices falls under the umbrella term of "adverse effect." Therefore, Choice C is not the correct answer, as the question asks for the best choice to describe what has occurred. Choice D is incorrect. A medication error is any preventable adverse drug event involving inappropriate medication use by a patient or health care professional (HCP). In this case, no evidence indicates a preventable adverse drug event occurred. As described above, an idiosyncratic reaction is an unpredictable and abnormal response to a medication. Here, the medication was administered per the HCP order. This reaction was unanticipated and unforeseeable and, therefore, not a medication error. Therefore, Choice D is incorrect. An idiosyncratic reaction is an abnormal and unexpected response to a medication, other than an allergic reaction, that is peculiar to an individual client. This type of reaction is nearly always unpredictable. There is a wide range of idiosyncratic drug reactions which may occur, such as maculopapular eruptions, eosinophilia, Stevens-Johnson syndrome, and exfoliative dermatitis. An allergic reaction (also known as a hypersensitivity reaction) involves the client's immune system. Immune system proteins (immunoglobulins) recognize the drug molecule, its metabolite(s), or another ingredient in a drug formulation and trigger an immune response. At that point, immunoglobulin proteins bind to the drug substance attempting to neutralize the drug. Various chemical mediators, such as histamine, cytokines, and other inflammatory substances are released. Physical manifestations of this response range from skin erythema or mild rash to severe, even life-threatening reactions such as the constriction of bronchial airways and tachycardia. This is considered a type of adverse drug event. Two types of adverse drug reactions are allergic reactions (often predictable) and idiosyncratic reactions (usually unpredictable). A medication error does not need to cause patient harm to be classified as a medication error.

At 2100, you administered a mildly sedating medication to your client per an order written by the attending health care provider (HCP) earlier today. Upon your reassessment of this client at 2200, you find the client restless, agitated, and hyperactive. Which of the following is the best choice to describe what has most likely occurred? A. An allergic reaction B. An idiosyncratic reaction C. An adverse effect D. A medication error

E, F Choices E and F are correct. Cryoprecipitate contains fibrinogen and is used for the treatment of clotting disorders. Platelets are used for the treatment of a bleeding disorder and a platelet deficiency disorder. Choice A is incorrect. Packed red blood cells (PRBCs) are used for several disorders, including anemia, post-operative blood replacement, and slow bleeding; not for a clotting disorder. Choice B is incorrect. Whole blood is reserved for only the most severe cases of bleeding and illness. Choice C is incorrect. Plasma transfusions are indicated for clients adversely affected with the need for blood volume expansion and depleted clotting factors; not for clients affected with anemia or low hematocrit. Choice D is incorrect. A plasma expander is used for hypovolemic circulatory shock and not for a platelet disorder. Plasma expanders include crystalloids and colloids. 0.9% NaCl and lactated ringers are examples of crystalloid plasma expanders. Albumin and gelofusin are examples of colloid plasma expanders. Choice G is incorrect. Albumin is indicated for clients adversely affected with the need for blood volume expansion and depleted plasma proteins; not for an excess of plasma proteins.

Select the blood product that is accurately paired with its indications for use in terms of the client's disorder or disease. Select all that apply. A. Packed red blood cells: A clotting disorder B. Whole blood: Relatively all cases of bleeding and hemorrhage C. Plasma: Anemia and low hematocrit D. A plasma expander: A platelet disorder E. Cryoprecipitate: A clotting disorder F. Platelets: A bleeding disorder G. Albumin: An excess of plasma proteins

A Choice A is correct. Due to their local action, intranasal sympathomimetics produce few systemic effects. However, one side effect associated with their use is rebound congestion. Prolonged use causes hypersecretion of mucus and worsened nasal congestion once the drug effects wear off. This rebound effect sometimes leads to a cycle of increased drug use as the condition worsens. Because of the risk of rebound congestion, intranasal sympathomimetics should be used for no longer than 3-5 days. Prolonged use of decongestant drops (3 to 5 days) can lead to rebound congestion, which is relieved by discontinuing the medication for 2 to 3 weeks. Nasal congestion results from dilation of nasal blood vessels due to infection, inflammation, or allergy. With this dilation, there is a transudation of fluid into the tissue spaces, resulting in swelling of the nasal cavity. Nasal decongestants (sympathomimetic amines) stimulate the alpha-adrenergic receptors, producing vascular constriction (vasoconstriction) of the capillaries within the nasal mucosa. The result is a shrinking of the nasal mucous membranes and a reduction in fluid secretion (runny nose). Decongestants can make a client jittery, nervous, or restless. These side effects decrease or disappear as the body adjusts to the drug. When nasal decongestants are used for longer than 5 days, instead of the nasal membranes constricting, vasodilation occurs, causing an increased stuffy nose and nasal congestion. The nurse should emphasize the importance of limiting the use of nasal sprays and drops. As with any alpha-adrenergic drug (for example, decongestants), blood pressure and blood glucose levels can increase. These drugs are contraindicated and should only be used with extreme caution for clients with hypertension, cardiac disease, hyperthyroidism, and diabetes mellitus. Choice B is incorrect. The nurse should instruct the patient to discontinue the nose drops. Additionally, the nurse should not tell a client which medications to take. Instead, this responsibility belongs to the doctor. Choices C and D are incorrect. A more potent decongestant is not needed. Instead, the patient should stop using the drops (Choice C). The frequency should not be increased (Choice D).

The client using over-the-counter nasal decongestant drops reports unrelieved and worsening nasal congestion. What is the appropriate instruction for this client? A. Discontinue the medication for several days B. Use a combination of oral decongestant medications and drops for better results C. Switch to a stronger dose of the decongestant drops D. Increase the frequency of the nasal decongestant drops

D Choice D is correct. Sleeping in a side-lying position ( "SOS," Sleep On Side) moves the fetus away from the inferior vena cava ( IVC). Therefore, the fetus's weight and pressure on the IVC are minimized, promoting venous return. Better venous drainage reduces potential lower extremity swelling, varicose veins, and hemorrhoids in the pregnant woman. Sleeping supine during pregnancy must be avoided because it increases the risk of late stillbirths. Traditionally, the left side-lying position is preferred over the right side because there is a theoretical risk of IVC compression due to the liver being present on the ride side. If the pregnant client is comfortable sleeping on the right side, it should not be discouraged because a 2019 meta-analysis has revealed both left and right-side lying positions are equally safe in pregnancy. Choice A is incorrect. Contrary to the statement, exercise promotes venous return when coupled with frequent rest periods. Pregnant women should be encouraged to exercise regularly. Choice B is incorrect. The client should avoid wearing knee-high stockings with garters. Garters are tight straps present at the top of the stockings to prevent the stockings from slipping down the leg. These garters can cause occlusion of the blood flow and increase the venous pressure in the extremities. The nurse must ensure that the stockings are not causing a garter effect at the knee or thigh level. The nurse should advise the pregnant client to wear graduated compression stockings ( GCS) or thrombo-embolus deterrent stockings ( TEDs). These should be applied up to above the point of varicose enlargement. In pregnancy, GCS or TEDs also serve as mechanical prophylaxis against deep vein thrombosis ( DVT). Choice C is incorrect. The client should increase her intake of vitamin C found in citrus fruits as vitamin C is involved in forming blood vessel collagen and endothelium. Additionally, vitamin C helps increase iron absorption in the gastrointestinal tract. Varicose veins are a common symptom during pregnancy, affecting about 40% of pregnant women. The gravid uterus puts pressure on the IVC, increasing venous pressure in the pelvis and lower extremities. In addition to the varicose veins in the lower extremities, varicosities in the vulvar region and hemorrhoids are also common in pregnancy. Women should be informed that these do not cause harm. Side-lying position and compression stockings can improve the varicose-related symptoms, but the evidence shows these measures will not prevent varicose veins from actually appearing.

The client, who is 24 weeks pregnant, is complaining to the nurse about her "worsening varicosities." The nurse would advise her to: A. Avoid exercise as blood pools in her legs during movement B. Wear knee-high hose with garters C. Avoid citrus fruits D. Sleep in a side-lying position

C Choice C is correct. Luteinizing hormone is the hormone chiefly responsible for the release of an ovum from a woman's ovary. Choice A is incorrect. Estrogen is responsible for creating a thick endometrium wall to protect an implanting fertilized egg. Choice B is incorrect. Testosterone is not responsible for releasing an ovum. Choice D is incorrect. Human chorionic gonadotropin is produced by the placenta and is not responsible for the release of an ovum.

The fertility nurse is providing education to a woman hoping to become pregnant. This nurse would be most correct in stating that which of the following hormones is chiefly responsible for the release of an ovum from a woman's ovary? A. Estrogen B. Testosterone C. Luteinizing hormone D. Human chorionic gonadotropin

C Choice C is correct. Nephroblastoma is a childhood cancer involving the kidney(s). Hypertension may occur because of the surge in renin triggered by the tumor. Choices A, B, and D are incorrect. Liver function tests may be monitored in the event of metastasis. However, an elevation of LFTs does not conclusively indicate metastasis. Capillary blood glucose and visual acuity are not routinely monitored for nephroblastoma. These two parameters are irrelevant to the condition. Nephroblastoma is a tumor affecting the kidney(s). The average age at diagnosis is three years in children with single kidney disease. It is slightly younger for those with bilateral involvement. Nephroblastoma nursing care involves ➢ Frequent blood pressure monitoring because this tumor may induce renin-related hypertension ➢ Avoid any activities that may cause palpation of the abdomen ➢ Gastrointestinal assessment as obstruction may consequently occur (absent bowel sounds, abdominal distention should be reported) ➢ Assessment of hemorrhage (tachycardia and hypotension)

The nurse is caring for a child with nephroblastoma. To prevent complications from this tumor, the nurse should closely monitor the client's A. liver function tests. B. capillary blood glucose. C. blood pressure. D. visual acuity.

A Choice A is correct. Trichotillomania is a syndrome that causes a client to engage in hair-pulling. This disorder is categorized as an obsessive-compulsive disorder. Common sites for hair pulling include the eyebrows, scalp hair, and chin. Selective serotonin reuptake inhibitors (SSRIs) combined with psychotherapy are effective treatments for this disorder. Medications that may be used include fluoxetine, citalopram, or paroxetine. Choices B, C, and D are incorrect. Trichotillomania is treated with SSRIs and psychotherapy. Antipsychotics are not routinely used; thus, haloperidol is excluded. Amphetamine is indicated in the treatment of ADHD, not obsessive compulsive related disorders such as trichotillomania. Bupropion is an atypical antidepressant that does not modulate serotonin, thus, is not used in the treatment of trichotillomania. Trichotillomania is a hair-pulling disorder that is categorized under obsessive-compulsive disorder. This condition may cause an individual to pull the hair on their scalp and eyebrows. Psychotherapy and SSRIs (fluoxetine, paroxetine, citalopram) may treat this disorder. The hair-pulling typically worsens when the client is under a period of stress. Trichotillomania may also occur with skin picking disorder. Both disorders are treated the same (psychotherapy and SSRIs).

The nurse is caring for a client diagnosed with trichotillomania. The nurse anticipates a prescription for which medication from the primary healthcare provider (PHCP)? A. Fluoxetine B. Amphetamine C. Haloperidol D. Bupropion

B Choice B is correct. A client with Cushing's disease has too many steroids and will have manifestations such as central obesity, weight gain, hypokalemia, hypernatremia, and hypertension. The client will not need more steroids during periods of stress as this is necessary for a patient with Addison's disease to prevent a crisis. Choices A, C, and D are incorrect. A client with Cushing's disease will need to eat more potassium-rich foods because of the potassium elimination associated with the high aldosterone levels. Excessive cortisol levels may cause immunosuppression thus, enabling opportunistic bacterial infections. The client should be instructed to do weight-bearing exercises because Cushing's may cause loss of bone density. High levels of cortisol and aldosterone characterize Cushing's disease. The client will have clinical features such as truncal obesity, hyperglycemia, hypernatremia, hypokalemia, and hyperlipidemia.

The nurse is caring for a client newly diagnosed with Cushing's disease. Which of the following client statements requires follow-up? A. "I will need to eat more potassium-rich foods." B. "I will need more steroids during periods of stress." C. "I will be at a higher risk for an infection." D. "I should do weight-bearing exercises."

C Choice C is correct. The client is demonstrating manifestations of a febrile reaction. A febrile reaction is usually not life-threatening and requires the nurse to pause the transfusion. The transfusion should be paused so the nurse may contact the PHCP and obtain a prescription for APAP, commonly prescribed for a client with this type of reaction. Choices A, B, and D are incorrect. The nurse should pause the transfusion because this will prevent further client discomfort. The nurse will need to execute these actions, but pausing the transfusion is essential.

The nurse is caring for a client receiving a unit of packed red blood cells (PRBCs). The client reports chills, and their oral temperature is 103° F (39.4° C). Which action should the nurse take first? A. Obtain a prescription for acetaminophen (APAP) B. Obtain blood cultures C. Pause the transfusion D. Notify the primary healthcare provider (PHCP)

C Choice C is correct. Cardiac catheterization involves the insertion of a large catheter into the femoral or radial artery to access the coronary arteries. A stent may be placed to keep the lumen of the artery open. This test can diagnose narrowing in the coronary arteries and intervene with angioplasty and stenting, if necessary. Choice A, B, D is incorrect. Exercise electrocardiography is commonly known as a stress test. This is a planned procedure that examines exercise tolerance and its cardiovascular effects. The client is having an acute infarction, and this test would be inappropriate. A chest CT may assist in diagnosing an occlusion in the coronary artery, but this test does not allow for intervention. An echocardiogram is performed to examine the heart's structures and its output. This is not appropriate during an acute myocardial infarction. When interventions needed, percutaneous coronary intervention is performed in the cardiac catheterization laboratory and combines clot retrieval, coronary angioplasty, and stent placement. Under fluoroscopic guidance, the cardiologist performs initial coronary angiography, inserting an arterial sheath and advancing a catheter retrograde through the aorta. Here the physician may determine which arteries are narrowed and require intervention. Intervention may come in the form of angioplasty with or without stenting.

The nurse is caring for a client with acute myocardial infarction (AMI). Which diagnostic intervention should the nurse anticipate? A. Exercise electrocardiography B. Computed tomography (CT) of the chest with contrast C. Cardiac catheterization D. Echocardiogram

C Choice C is correct. Heat should not be applied to the abdomen of patients experiencing pain from appendicitis. Heat will cause vasodilation and increased blood flow to the appendix which may lead to rupture. A ruptured appendix puts the client at risk for a life-threatening condition known as peritonitis. Choices A, B, and D are incorrect. Applying ice packs to a patient's abdomen experiencing discomfort related to appendicitis is an appropriate non-pharmaceutical intervention. Using breathing techniques to work through appendicitis pain is an appropriate non-pharmaceutical intervention. Encouraging plenty of rest is an excellent way to prevent and manage pain from appendicitis.

The nurse is caring for a client with appendicitis experiencing pain. Which pain relief method would be inappropriate for this client? A. Applying ice packs to the abdomen B. Practicing breathing exercises with the patient C. Using a heating pad D. Encouraging rest

B Choice B is correct. A transvaginal ultrasound is a gold standard in diagnosing placenta previa. A transvaginal ultrasound is more accurate than transabdominal ultrasound in determining the location of the placenta. Please recognize that the vagal ultrasound probe can safely be inserted a few inches from the cervix to capture the placenta alteration. Any advancement of the probe into the cervix could cause bleeding. However, if done according to the recommendations, the appropriate angle between the cervix and the vaginal ultrasound probe prevents the probe from accidentally moving into the cervical canal. Choices A, C, and D are incorrect. A manual cervical examination is contraindicated in placenta previa because this may trigger bleeding. A manual cervical exam attempts to invade the cervical canal directly, whereas a transvaginal ultrasound probe is inserted away from the cervix. While placenta previa is not an absolute contraindication for a contraction stress test, it raises the risk of bleeding. A nonstress test is safe and often performed for individuals with placenta previa. However, this test does not confirm placenta previa; instead, it evaluates fetal wellbeing.

The nurse is caring for a client with suspected placenta previa. The nurse anticipates a prescription for which diagnostic test to confirm this finding? A. Manual cervical exam B. Transvaginal ultrasound C. Contraction stress test D. Nonstress test

D Choice D is correct. A client receiving oxygen via non-rebreather is receiving approximately 80%-95% Fio2. If the best oxygen saturation is 92%, this is concerning and may warrant more aggressive measures to improve oxygen saturation. Choices A, B, and C are incorrect. A client admitted with an asthma exacerbation would have manifestations such as tachypnea and wheezing. This is not a priority because the prescribed treatment is being administered. A concern would be if the wheezing does not improve or if the sudden cessation of wheezing should occur. A client with pulmonary emphysema should eat while using a nasal cannula as this increases the oxygen demand. A client with pneumonia should be encouraged to ambulate around the nursing unit. The appropriate PPE is applied to this client, which is a surgical mask. Nonrebreather is an oxygen delivery device that may deliver the highest oxygen level in the low-flow systems. This device features a reservoir bag that allows the client to draw the needed oxygen. Additionally, flaps prevent room air from entering the exhalation ports. If a client is not responding favorably to this therapy, the primary healthcare provider (PHCP) may consider intubation and mechanical ventilation. When nonrebreather therapy is initiated, the oxygen delivery device should be set at 10 to 15 L/min to ensure that the bag is kept inflated.

The nurse is caring for a group of clients. It is a priority to follow up on which client situation? A. A client admitted with an asthma exacerbation that is wheezing while receiving albuterol via nebulizer. B. A client admitted with pulmonary emphysema who puts on their nasal cannula oxygen before eating. C. A client with pneumonia is ambulating around the nursing unit while wearing a surgical mask. D. A client receiving oxygen via nonrebreather and has an oxygen saturation of 92%.

C Choice C is correct. This patient's symptoms are consistent with Raynaud's phenomenon. This condition is characterized by sudden or intermittent vasospasms in the fingertips and toes in response to cold temperatures or stress, resulting in decreased blood flow and blanching of the skin in these areas. This phase is typically followed by a blue phase, when hemoglobin releases oxygen into these tissues, and a red phase, when the areas are rewarmed. Sensations of tingling and numbness are common during these episodes. Choice A is incorrect. Sjögren's syndrome describes a condition of dry eyes and dry mouth that is experienced by approximately 20% of patients with scleroderma. Choice B is incorrect. Sclerodactyly describes the tightening of the skin of fingers and toes. Choice D is incorrect. Telangiectasia describes the presence of red spots on the hands, forearms, palms, face, and lips due to capillary dilation.

The nurse is caring for a patient with a medical diagnosis of scleroderma who reports fingertips tingling and turning white in response to cold or stress. The nurse would recognize these symptoms as which problem? A. Sjögren's Syndrome B. Sclerodactyly C. Raynaud's phenomenon D. Telangiectasia

D Choice D is correct. NG tubes should be watered every 4 hours with 30 - 50 mL of water or normal saline. Choices A, B, and C are incorrect. These are the inaccurate measurements necessary.

The nurse is caring for a patient with a nasogastric tube. Irrigation should be performed every 4 hours to assess for NG tube patency. The nurse should instill how many milliliters of water or normal saline? A. 15 - 25 mL B. 20 - 30 mL C. 20 - 40 mL D. 30 - 50 mL

B Choice B is correct. This patient should have a bag-valve-mask setup (Ambu) and suction equipment at the bedside. Recent total thymectomy procedure puts this patient at risk for myasthenic crisis (an exacerbation of myasthenia gravis symptoms due to insufficient cholinergic medications) and risk for impaired gas exchange (due to potential hemothorax/pneumothorax). A myasthenic crisis results in muscle weakness and increases the risk of respiratory issues. Noninvasive mechanical ventilation should be used to support respiratory status and improve gas exchange in the event of respiratory distress until other interventions are available. Choice A is incorrect. It would not be necessary to keep a crash cart at the bedside for this patient but it should be readily available on the unit in case of an emergency. Choice C is incorrect. Myasthenia gravis causes muscle weakness in the face and mouth, so it may be difficult for this patient to form a seal around the mouthpiece of an incentive spirometer. In addition, diaphragm and intercostal muscles may be weakened, which makes it difficult to deep breathe or cough. If the patient is able to use an incentive spirometer it would be appropriate to have it at the bedside, but would not be the highest priority, as it would only help to prevent atelectasis after surgery, not improve potential acute respiratory distress. Choice D is incorrect. The Tensilon test is used to differentiate cholinergic crisis vs myasthenic crisis and may cause cardiac dysrhythmias or cardiac arrest. Atropine, the antidote for Tensilon, should be available if the patient is undergoing this test, but would not be important following a thymectomy.

The nurse is caring for a patient with myasthenia gravis who is recovering from a total thymectomy. Which would be the highest priority to have at this patient's bedside? A. Crash cart B. Bag-valve mask C. Incentive spirometer D. Atropine sulfate

A, B, C, D Choices A, B, C, and D are correct. The nurse arranging for an interdisciplinary conference, consulting with case management, initiating outpatient referrals, and performing post-discharge phone calls are all relevant to effective care coordination. These actions work to improve care delivery through effective communication with other members of the healthcare team. Choice E is incorrect. While appropriate transmission-based precautions are important to implement and maintain, they are not relevant to care coordination, focusing on organizing client care, and sharing information with pertinent individuals involved in the patient's care. Effective care coordination involves providing organized, cost-effective care involving multiple disciplines through good communication. Effective care coordination aims to reduce costs by decreasing hospitalization and mitigating errors through linear communication.

The nurse is caring for assigned clients. Which of the following actions would reflect effective care coordination? Select all that apply. A. Arranging for an interdisciplinary conference B. Consulting with case management for a discharge plan C. Initiating appropriate outpatient referrals D. Performing post-discharge phone calls E. Implementing transmission-based precautions

A, B Choices A and B are correct. The hypoglossal cranial nerve (XII) is central to the skeletal muscles of the tongue and assists with swallowing. If a client has an impairment of this cranial nerve, aspiration precautions should be implemented. These precautions include observing the client during meals and having patent suction at the bedside. Choices C, D, and E are incorrect. The hypoglossal nerve has no relevance in vision. Thus, providing large print education materials, teaching the client to scan the room, and alternating the use of an eye patch are not relevant to this cranial nerve. An eye patch is an effective intervention for a client experiencing diplopia. Aspiration precautions involve collaborating with the registered dietician and/or speech therapist to determine which would be appropriate. In general, aspiration precautions involve supervised mealtimes, having the head of the bed elevated during a meal, thickening the liquids as directed, and coaching the client through swallowing. If the nurse is feeding a client with aspiration precautions, the nurse should not mix any food consistencies and always have oral suction available.

The nurse is developing a plan of care for a client with an impairment to the hypoglossal cranial nerve. Which of the following should the nurse include in the client's plan of care? Select all that apply. A. Observe the client during meals B. Keep suction at the bedside C. Provide large print education materials D. Teach the client to scan the room E. Alternate the use of an eye patch

B Choice B is correct. A child should walk their bike through busy intersections to reduce their risk of being hit by an automobile. Choices A, C, and D are incorrect. These statements require follow-up because they do not indicate effective teaching. When riding a bicycle, it should be directed with the traffic flow. Any individual riding a bicycle should always wear a helmet, no matter the distance they plan on riding. To avoid serious foot injuries, the child should always wear proper fitting shoes while bicycling. Always wear a properly fitted helmet that the US Consumer Product Safety Commission approves; replace a damaged helmet. • Ride bicycles with traffic and away from parked cars. • Ride single file. • Walk bicycles through busy intersections and at crosswalks. • Give hand signals well in advance of turning or stopping. • Keep as close to the curb as practical. • Watch for drain grates, potholes, soft shoulders, and loose dirt or gravel. • Keep both hands on handlebars except when signaling. • Never ride double on a bicycle. • Do not carry packages that interfere with vision or control; do not drag objects behind bike. • Watch for and yield to pedestrians. • Watch for cars backing up or pulling out of driveways; be especially careful at intersections. • Look left, right, then left before turning into traffic or roadway. • Never hitch a ride on a truck or other vehicle. • Learn the rules of the road and respect for traffic officers. • Obey all local ordinances. • Wear shoes that fit securely while riding. • Wear light colors at night, and attach fluorescent material to clothing and bicycle. • Be certain the bicycle is the correct size. • Equip the bicycle with proper lights and reflectors. • Have the bicycle inspected to ensure good mechanical condition. • When riding as a passenger, wear appropriate-size helmet and sit in a specially designed protective seat.

The nurse is educating the parents of a child who plans on riding their bicycle. Which statements, if made by the parents, indicate effective understanding? A. "I should tell my child should ride against the traffic pattern." B. "I should instruct my child to walk their bike through busy intersections." C. "Wearing a helmet is only necessary when my child is riding near a busy intersection." D. "My child can ride their bike barefoot as long as it's short distances."

C Choice C is correct. The nurse should make sure that the patient receives a prescribed bronchodilator about 15 minutes before their chest physiotherapy procedure. Chest physiotherapy is used to loosen secretions trapped in the lungs. When administered before this procedure, a bronchodilator helps to dilate the bronchioles and liquify secretions. Choice A is incorrect. A gown or piece of fabric should be placed between the hands or percussion device right before the procedure. However, this should be done just before the process. Another option (administering bronchodilator 15 minutes prior) exists in the choices and is the initial action. Choice B is incorrect. Walking with the patient before the procedure is not necessary before chest physiotherapy. Choice D is incorrect. Calling the physician to confirm the x-ray results is not necessary at this time and does not alter the plan for chest physiotherapy.

The nurse is planning to assist a respiratory therapist in performing a chest physiotherapy procedure. Which of the following is the initial action by the nurse before the process? A. Place a gown or fabric between the hands or percussion device and the client's skin B. Walk with the patient for a few laps around the unit to aid in percussion C. Administer a prescribed bronchodilator D. Call the physician to confirm x-ray results

A, B, E Choices A, B, and E are correct. Choices A and B. Hypercalcemia can occur in various conditions such as primary hyperparathyroidism, malignancies, milk-alkali syndrome, medications, vitamin D toxicity, and sarcoidosis. Symptomatic hypercalcemia can lead to constipation, psychosis, polyuria, and dehydration. Clients with hypercalcemia should take some dietary precautions to reduce calcium intake. Broccoli is rich in calcium and should therefore be avoided in clients with hypercalcemia. Milk is rich in calcium and should therefore be avoided in clients with hypercalcemia. Choice E. Vitamin D is one substance that, along with parathyroid hormones, regulates a person's calcium levels. Several kinds of seafood are rich in Vitamin D and should be avoided if hypercalcemia is a concern. Choices C and D are incorrect. Whole wheat pasta is not a calcium-rich food. Bananas are particularly high in potassium, not calcium. The nurse does not need to instruct the client with hypercalcemia to avoid whole wheat pasta or bananas.

The nurse is reviewing dietary teaching with a client who has hypercalcemia. Which foods should the nurse recommend that the client avoid? Select all that apply. A. Broccoli B. 2% milk C. Whole wheat pasta D. Bananas E. Seafood

A Choice A is correct. An early sign of illness involves an increase in the patient's heart rate. Tachycardia is a rapid response to hypovolemia. A heart rate change from 80 to 125 bpm warrants further investigation into a possible illness. Choice B is incorrect. A blood pressure change from 125/90 to 119/82 mmHg is not a significant drop enough to indicate hypovolemia or considerable hemorrhage. Shock is defined as mean arterial pressure less than 65 mmHg or systolic blood pressure less than 90 mmHg. Choice C is incorrect. A respiratory rate decrease from 22 to 16 breaths per minute is an expected finding after delivery and is not alarming. Moreover, significant bleeding or hemorrhagic shock is associated with tachypnea (increase in the respiratory rate), not a decrease. Choice D is incorrect. Some perineal soreness is normal after delivery and does not necessarily indicate impending hemorrhage.

The postpartum nurse is monitoring a new mother for signs of illness following vaginal delivery of a newborn infant. Which of the following is an early sign of excessive blood loss? A. Heart rate change from 80 to 125 bpm B. Blood pressure change from 125/90 to 119/82 mmHg C. A decrease in respiratory rate from 22 to 16 breaths per minute D. Reports of perineal soreness

C Choice C is correct. Assessing if the patient is responsive is the primary concern of the nurse in this example. Choices A and B are incorrect. The client's responsiveness is a priority before moving the client. Choice D is incorrect. This answer choice would be the least important among the choices given.

Upon entering a client's room, the nurse finds the client lying on the floor. What is the first action the nurse should implement? A. Call for help to get the client back in bed B. Assist the client back to bed C. Establish if the client is responsive D. Ask the client what happened

Keep the child calm and quiet Help the child to sit up and lean forward Apply pressure to the nose for at least 10 minutes If still bleeding, Insert cotton into each nostril Apply ice to the bridge of the nose The priority nursing action to take is to keep the child calm and quiet. If the child becomes distressed and is crying, it will exacerbate the bleed. Next, the nurse needs to sit the child up and lean them forward. Many parents think they should pinch the child's nose and tilt their head backward, but this will not aid in stopping the bleed and can be an aspiration risk. Do not let the parent tilt the child's head back. Next, begin applying pressure to the nose, and check to see if the bleeding continues after 10 minutes. If the nose is still bleeding, the next action would be to insert absorbent cotton into each nostril. If the nose continues to bleed after that, the following priority action is to apply ice to the bridge of the nose to aid in vasoconstriction thus stopping the bleed.

While working on a pediatric floor, your 2-year old patient begins experiencing epistaxis. Place the appropriate priority nursing actions in the correct sequence: Keep the child calm and quiet Help the child to sit up and lean forward Apply pressure to the nose for at least 10 minutes Apply ice to the bridge of the nose If still bleeding, Insert cotton into each nostril

C Choice C is correct. The CDC and experts at the American Academy of Pediatrics agree that there is no credible evidence that the MMR vaccine causes autism spectrum disorder (ASD). Some of the concerns may be because children typically get the MMR vaccine at about the same time that signs of ASD appear. In fact, in 2013, the CDC conducted a study that showed that vaccines do not cause ASD. This study showed that the antigens in vaccines that produce antibodies were the same between children diagnosed with ASD and children without ASD. Currently, there are no MMR vaccines that contain mercury. Although there is no evidence that mercury causes ASD, mercury was removed from all childhood vaccinations by 2001. The exception to that is that some multi-dose flu vaccines may still contain traces of thimerosal (a chemical containing mercury).

You are educating a mother about the association between autism and the MMR vaccine. You know that the mother understands your instructions when she says: A. "My child should not get the vaccine since it is known to cause autism." B. "My child should get the individual immunizations for measles, mumps, and rubella since the individual vaccines do not cause autism." C. "My child should get the MMR immunization since there is no evidence that it causes autism." D. "My child should not get the immunization because it contains mercury."

A, C Choices A and C are correct. Any changes in the bowel and bladder habits for a patient are concerning for cancer and clients should be taught to report this to the healthcare provider immediately (Choice A). Any difficulties swallowing are potential early signs of cancer that should be reported (Choice C). Choice B is incorrect. Weight loss, not gain, is concerning for cancer. Choice D is incorrect. Fatigue, not insomnia, is the most common symptom of cancer. For clients experiencing unexplained fatigue, their healthcare provider should be notified.

You have been asked to lead a health promotion course for the community about cancer. Which of the following symptoms should you include in your education about early signs and symptoms of cancer? Select all that apply. A. Incontinence B. Weight gain C. Trouble swallowing D. Insomnia

A Choice A is correct. This client with chronic pancreatitis and gastroparesis is complaining of a migraine headache. Butorphanol is available in the oral form, transnasal form, transdermal, and parenteral form. The doctor has ordered butorphanol orally as needed for pain, but you would call the doctor and suggest transnasal butorphanol because the client has gastroparesis. Choice B is incorrect. Butorphanol is not available for rectal administration. Choice C is incorrect. You would not administer butorphanol orally for pain because this route is contraindicated among clients with gastroparesis. Choice D is incorrect. You cannot administer transdermal butorphanol without a doctor's order. The doctor needs to be called to obtain such an order.

Your client, who has chronic pancreatitis and gastroparesis, is complaining of a migraine headache. The doctor has ordered butorphanol orally as needed for pain. What would you do? A. Call the doctor and suggest transnasal butorphanol because the client has gastroparesis. B. Call the doctor and suggest rectal butorphanol because the client has pancreatitis. C. Administer the butorphanol orally as ordered. D. Administer the butorphanol transdermally for pain.

B Choice B is correct. The nurse should suspect a possible ectopic pregnancy. Abdominal pain, vaginal bleeding, and an adnexal mass are the classic triad for an ectopic pregnancy. The developing chorion produces progesterone. A normal progesterone level is > 15 ng/mL. A lower than normal progesterone level is uncommon in normal pregnancies but is very common in an ectopic pregnancy. Further testing will usually be done to confirm the diagnosis..

Your new client presented with a positive home pregnancy last night. She has abdominal pain, some vaginal bleeding. and you note an adnexal mass on palpation. You order a progesterone level, which returns as 13 ng/mL. Your initial impression is: A. Early normal pregnancy B. Possible ectopic pregnancy C. Abnormal intrauterine pregnancy D. Incorrect home pregnancy test

B Choice B is correct. Gout is a disease that develops when high uric acid levels form crystals that accumulate in joints. Allopurinol is a medication commonly prescribed to decrease uric acid levels. Choices A, C, and D are incorrect. Colchicine, naproxen, or prednisone may be used during an acute gouty attack. The primary intention of these medications is to decrease the overall inflammation with the gouty attack. Gout is a condition characterized by increased uric acid Primary gout results from a disorder of purine metabolism Secondary gout involves excessive uric acid in the blood caused by another disease Painful episodes may be triggered by dehydration or non-adherence to the low-purine diet Nursing care is focused on fluid repletion, avoiding therapeutic touch, and administering prescribed anti-inflammatories Client education focuses on appropriate hydration and reducing the intake of purines

The nurse is caring for a client newly diagnosed with gout. The nurse anticipates a prescription for which medication? A. Colchicine B. Allopurinol C. Naproxen D. Prednisone

A, C Choices A and C are correct. For a nonstress test to be reactive, there must be two accelerations. Acceleration is defined as an increase in fetal heart rate by 15 beats per minute for at least 15 seconds with movement (Choice A). For a nonstress test to be reactive, there must be two accelerations. Acceleration is defined as an increase in fetal heart rate by 15 beats per minute for at least 15 seconds with movement (Choice C). Choice B is incorrect. Any decrease in fetal heart rate is a deceleration, which is an indicator of fetal distress and a nonreassuring sign. Slowdowns would lead to a nonreactive nonstress test. Choice D is incorrect. Any decrease in fetal heart rate is a deceleration, which is an indicator of fetal distress and a nonreassuring sign. Slowdowns would lead to a nonreactive nonstress test.

Which of the following are required for a nonstress test to be considered reactive? Select all that apply. A. Two increases in the fetal heart rate of 15 beats per minute. B. Two decreases in the fetal heart rate of 15 beats per minute. C. Two increases in the fetal heart rate for 15 seconds. D. Two decreases in the fetal heart rate for 15 seconds

B Choice B is correct. This ABG (Arterial Blood Gas) shows respiratory acidosis. The pH is less than 7.35, which is acidotic. The PCO2 is higher than 45, which is acidotic. Lastly, HCO3 is greater than 26, which is alkalotic. This occurs as compensation to respiratory acidosis, where kidneys try to conserve bicarbonate to bring pH back to normal. The PCO2 shows acidosis like the pH, so we know that this is respiratory acidosis.

Which of the following is the correct interpretation for the following arterial blood gas? pH: 7.27 PCO2: 55 HCO3: 28 A. Metabolic acidosis B. Respiratory acidosis C. Metabolic alkalosis D. Respiratory alkalosis

C Choice C is correct. The nursing intervention that is necessary after the placement of this epidural catheter and the initiation of an opioid epidural infusion is to monitor the client for signs of respiratory depression and level of sedation at least every hour for the first 24 hours. Choice A is incorrect. Ensuring the availability and immediate accessibility of Actiq to reverse any respiratory depression is not an appropriate nursing intervention. Actiq is a short-acting opiate that is used to treat breakthrough pain and will only increase respiratory depression, not reverse it. Choice B is incorrect. Ensuring the availability and immediate accessibility of Sublimaze to reverse any respiratory depression is not an appropriate nursing intervention. Sublimaze is an opioid medication and will only increase respiratory depression, not reverse it. Choice D is incorrect. Although the nurse must monitor the client carefully, this monitoring of the client for any signs of respiratory depression and level of sedation is inaccurate.

Your client has just got an epidural catheter to manage their severe, continuous pain. Which of the following is a nursing intervention that is necessary after the placement of this epidural catheter and the initiation of an opioid epidural infusion? A. Ensure the availability and immediate accessibility of Actiq to reverse any respiratory depression. B. Ensure the availability and immediate accessibility of Sublimaze to reverse any respiratory depression. C. Monitor the client at least every hour for the first 24 hours for any signs of respiratory depression and level of sedation. D. Monitor the client at least every 2 hours for the first 24 hours for any signs of respiratory depression and level of sedation.

B Choice B is correct. This client with sickle cell crisis has a high risk of cerebrovascular accidents (CVA). Since the client has a severe headache, it is best to rule out a CVA before initiating all other interventions. Choice A is incorrect. Giving oxygen can help reduce the cells' sickling, but this is not the first intervention for the client's headache. Furthermore, oxygen-carrying capacity is reduced when the cells are actively sickling. Increasing oxygen content in the blood will not significantly improve the oxygen-carrying capacity in a non-hypoxemic sickle cell client. There is no indication that the client is hypoxemic. (Note: If the information is absent in the question stem, that vital is considered normal. When evaluating the questions, do not add additional information to the question stems on the NCLEX). Choice C is incorrect. The nurse must first assess to determine whether the pain is from what is expected with the disease process or whether it is a complication. Administering pain medications right away would mask the actual disease process. Choice D is incorrect. Hydration can help in decreasing the sickling of cells. Choice D indicates that IV hydration is already in place. In SCD, the client should be kept euvolemic. Hypervolemia should be avoided because it can cause additional problems that come from fluid overload. Increasing the IV hydration without assessing the volume status is not the first intervention for this client

A 12-year-old is diagnosed with a vaso-occlusive sickle cell crisis and complains of severe headaches. What should be the nurse's initial intervention? A. Give oxygen at 6 liters per minute via nasal cannula. B. Assess the client's neurologic status. C. Give an intravenous dose of morphine. D. Increase the client's IV rate.

D Choice D is correct. Headache is one of the most common side effects of theophylline. It is essential to understand the difference between a side effect and drug toxicity- a side effect is something that can occur at a usual recommended dosage. On the contrary, drug toxicity (adverse drug event) occurs when there is overdosage or significant drug accumulation in the body above the therapeutic range. Common side effects of theophylline include headache, restlessness, nausea, and sleeplessness. On the other hand, the clinical manifestations of theophylline drug toxicity/ theophylline poisoning include cardiac dysrhythmias (presenting as palpitations, cardiac arrest), hypotensive shock, seizures/ status epilepticus, and refractory vomiting. Choice A is incorrect. Seizures are a sign of toxicity from theophylline, not just a common side effect. Choice B is incorrect. Palpitations and arrhythmias are a sign of theophylline drug toxicity as well. Choice C is incorrect. Hypotension and shock are seen with theophylline overdose/ drug toxicity and is not common side effect.

A 14-year-old is admitted to the medical ward for status asthmaticus. He was put on IV theophylline. Which manifestation would the nurse consider as a side effect of the drug? A. Grand mal seizures B. Severe palpitations C. Hypotension D. Headache

A Choice A is correct. A lumbar puncture (LP) reduces a client's cerebrospinal fluid volume and pressure. As a result of this decreased volume and pressure, headache results. This post-lumbar puncture headache is a common post-procedure complication, usually occurring hours to one to two days following the procedure, with severity ranging from moderate to severe. Hydration is a primary treatment for post-lumbar puncture headache. Increasing the client's fluid intake would facilitate the restoration of the client's cerebrospinal fluid volume. Choice B is incorrect. The administration of antihypertensive medications will not address the issue at hand. Currently, the client is experiencing a post-lumbar puncture headache due to decreased cerebrospinal fluid volume and pressure. Antihypertensive medications will not increase the volume or pressure of the cerebrospinal fluid. If the antihypertensive medications affect the cerebrospinal fluid, the medications will decrease the pressure, worsening the client's headache. Choice C is incorrect. Roll lenses act as a blindfold to reduce light irritation to the eyes for a client experiencing photophobia. Although this may benefit some clients, this is not the most appropriate nursing intervention. Choice D is incorrect. Administering antihypertensives will not address the problem. Placement of a cool pack on the puncture site may help temporarily reduce pain or inflammation, but this nursing intervention does not address the primary issue causing the client's headache. Therefore, this is not the most appropriate nursing intervention. Additional first-line treatments for post-lumbar puncture headache include recumbency, intravenous caffeine administration, and/or analgesic administration (as needed). Younger clients with a small body mass are at the most risk for a post-lumbar puncture headache. The amount of cerebrospinal fluid volume removed and duration of recumbency following the lumbar puncture do not affect the incidence of post-lumbar puncture headache. If the client's symptoms persist for more than 24 hours, the health care provider (HCP) should consider ordering an epidural blood patch (i.e., injection of a few mL of the client's clotted venous blood into the lumbar epidural space).

A lumbar puncture was performed on a client for a myelogram. After the procedure, the client complains of a severe headache. The most appropriate nursing intervention is: A. Increase the client's oral fluid intake B. Administer the prescribed antihypertensives to the client C. Give the client roll lenses D. Place a cool pack over the lumbar puncture site

D Choice D is correct. This client is displaying classic signs and symptoms of hypocalcemia (i.e., paresthesia and tetany). If left untreated, symptoms may progress to seizures, encephalopathy, and heart failure. More convincingly, the client's recent thyroidectomy supports a presumptive diagnosis of hypocalcemia. Although the thyroid gland in and of itself does not regulate calcium levels within the body, four parathyroid glands (responsible for releasing parathyroid hormone (PTH) to control calcium levels in your blood) are located within the thyroid. Hypoparathyroidism often results after the accidental removal of or damage to one or more parathyroid glands during thyroidectomy. The nurse should anticipate administering intravenous calcium gluconate to this client. Choice A is incorrect. Thyroid supplements are typically given post-thyroidectomy but are not directly related to this problem. Choice B is incorrect. Antispasmodics will not treat the cause of the problem. Choice C is incorrect. Barbiturates are not indicated in this situation and will not address the issue at the core of the problem. Permanent hypoparathyroidism occurs after < 3% of such thyroidectomies performed by experienced surgeons. Treatment guidelines typically include administering intravenous calcium gluconate to clients experiencing tetany. Clients who are not experiencing tetany are typically treated with oral calcium supplements.

A nurse is assigned to care for a client who recently underwent a thyroidectomy. The nurse notes that the client has developed peripheral numbness and tingling, muscle twitching, and spasms. Based on this information, the nurse should anticipate administering: A. Thyroid supplements B. Barbiturates C. Antispasmodics D. Intravenous calcium gluconate

C Choice C is correct. Chemotherapy agents place clients at increased risk of infection due to immune suppression of the medication(s), specifically by decreasing neutrophils. Neutropenia, a reduction in the blood neutrophil count, is common in chemotherapy clients. As the neutropenia increases, so does the client's risk (and severity of) bacterial and fungal infections. If a bacterial or fungal infection does occur, the likelihood of the infection spreading to other parts of the body increases. In a urinalysis, the presence of leukocytes and nitrites is indicative of a urinary tract infection. This result should indicate to the nurse that a urinary tract infection is present in this immunocompromised client, warranting the nurse to notify the HCP of the result so antibiotic therapy may be initiated immediately. Choice A is incorrect. An elevated amylase result in a client diagnosed with acute pancreatitis is an anticipated finding and would not warrant reporting the result to the HCP. Choice B is incorrect. In a client diagnosed with a septic leg wound, an elevated white blood cell count (also known as leukocytosis) is an anticipated finding. Leukocytosis usually occurs in response to infection, trauma, or inflammation. Since this client is known to be septic, the leukocytosis is an anticipated finding and, therefore, does not warrant the nurse immediately reporting this lab result to the HCP. Choice D is incorrect. The client's serum glucose level of 235 mg/dL is above the normal range of 70-110 mg/dL, but this is a relatively common finding in clients with type I diabetes mellitus and does not necessitate immediate reporting to the HCP. Neutropenia predisposes the client to bacterial and fungal infections. The risk of infection is proportional to the severity of neutropenia, with clients with neutrophil counts < 500/mcL (< 0.5 × 109/L) at the most significant risk. Febrile neutropenic clients are typically treated with broad-spectrum antibiotics pending definitive identification of the infection. Antibiotic prophylaxis may be indicated for some high-risk clients.

A nurse receives laboratory results for several clients under her care. Which client result would the nurse report to the health care provider (HCP) immediately? A. An elevated amylase result in a client diagnosed with acute pancreatitis B. An elevated white blood cell (WBC) count in a client with a septic leg wound C. A urinalysis positive for leukocytes and nitrites in a chemotherapy client D. A serum glucose of 235 mg/dL in a client with type 1 diabetes mellitus

A Choice A is correct. Patients at risk for aspiration should be encouraged to swallow with their chin down. The nurse would need to intervene and give direct instructions on how to chew and swallow to prevent aspiration. The risk for aspiration is high when any patient has increased chances of secretions, solids, or fluids entering the tracheobronchial passages. Following physician orders and ensuring the patient's food is at the ordered consistency is crucial. Nursing assistants and other ancillary personnel who may feed the client should be instructed on the proper way to feed, including allowing the patient to take their time and ensuring all food is swallowed before offering another bite. Fluids should be held until the end of the meal, when possible. Choices B, C, and D are incorrect. None of these require immediate nursing intervention.

A nursing assistant is feeding a patient with Parkinson's disease who is on aspiration precautions. Which action would require immediate intervention by the nurse? A. The nursing assistant reminds the client to keep his head back when he chews and swallows. B. The nursing assistant maintains the thickened liquid diet as ordered by the physician. C. The nursing assistant waits for the patient to finish swallowing before offering another bite. D. The nursing assistant does not offer fluids until the end of the meal.

C Choice C is correct. Grapefruit can interfere with other drugs, as well, including statins (atorvastatin, lovastatin, simvastatin), some antibiotics, and some cancer drugs. Grapefruit and its juice contain furanocoumarins, which block the enzymes that are involved in metabolizing many drugs, including calcium channel blockers. Medication blood levels can increase, resulting in toxicity. The levels of calcium channel blockers are increased when grapefruit or grapefruit juice is consumed, potentially causing hypotension. Choices A, B, and D are incorrect. Neither of these answer options adversely affect calcium channel blockers.

A patient is prescribed a calcium channel blocker to treat primary hypertension. When teaching the patient about the medication, which of these foods will the healthcare provider advise the patient to avoid? A. Eggs B. Milk C. Grapefruit D. Bananas

A Choice A is correct. Reposition the infant to open the airway while ensuring that you have a good seal with the mask on the newborn's face. Following that action, a team member should suction the infant's mouth and nose. Until the team establishes sufficient ventilation, there is no indication to increase oxygen concentration or begin CPR. The AHA and AAP focus on positive-pressure ventilation as the single most crucial step in the resuscitation of the newborn. Choices B, C, and D are incorrect.

A woman is in the labor and delivery suite at 37 weeks gestation. She has been under her obstetrician's care for preeclampsia. The labor nurse notices that the fetus is experiencing heart rate decelerations. You are part of the neonatal resuscitation team that responds to the call from the labor room nurse. The infant is born but does not respond to tactile stimulation. The group moves the infant to the warmer. You evaluate the infant and confirm he is still not breathing. You begin positive pressure blowing with room air. Another team member notes that the heart rate is 72 bpm and the newborn's chest is not moving with PPV on room air. The next appropriate action is to: A. Reposition the infant to open the airway B. Begin CPR C. Suction the infant with a bulb syringe D. Increase the oxygen concentration

A, C, D, F Choice A is correct. Anuria can occur as the result of end-stage renal failure as well as a severe shock when the perfusion of the kidneys is impaired. Choice C is correct. Oliguria can result from dehydration as well as other causes such as hypotension and a decrease in terms of the client's fluid intake. Choice D is correct. Polyuria can occur secondary to chronic nephritis and excessive fluid intake among other causes such as diabetes mellitus and diabetes insipidus. Choice F is correct. Dysuria can occur as a result of any urinary tract infection and other causes such as trauma. Choices B and E are incorrect: Choice B is incorrect. Urinary retention, rather than anuria, can occur as a result of the effects of general anesthesia. Choice E is incorrect. Dysuria does not occur as a result of stress. Dysuria, however, can occur as the result of a urinary tract infection and other causes such as trauma.

Select the urinary elimination problems that are accurately paired with its possible etiology. Select all that apply. A. Anuria: End stage renal failure B. Anuria: The effects of general anesthesia C. Oliguria: Dehydration D. Polyuria: Chronic nephritis E. Dysuria: Stress F. Dysuria: Urinary tract infection during pregnancy

D Choice D is correct This observation requires follow-up because a belt restraint should be applied to the client's waist - not the chest. Having a belt restraint secured over the client's chest is inappropriate. Choices A, B, and C are incorrect. These observations do not require follow-up. An indwelling catheter tubing should hang freely from the bed frame (and not from the side rails). The tubing should be without any kinks or loops. If a client has right-sided weakness, their ambulation device (cane, walker, etc.) should be placed on their stronger side. A client with a history of falling is at an increased risk for future falls, and placing a bedside commode is a measure to reduce falls (by decreasing the client's distance to ambulate to the bathroom). Belt restraints may be warranted for confused or impulsive clients who are continually trying to get out of bed or a chair after repeated redirection, when it's unsafe for them to get up unassisted. Belt restraints should be applied over clothing and secured over the client's waist. The restraint should be anchored to an immovable part of the bed or the chair.

The charge nurse is performing safety rounds on clients in the nursing unit. Which observation requires follow-up? A client with A. an indwelling urinary catheter hanging from the bed frame. B. right-sided weakness with their cane on the left side of the bed. C. a history of falling given a bedside commode. D. a belt restraint was applied and secured over the chest.

C Choice C is correct. The client is suffering from morphine toxicity. The nurse needs to administer the antidote, which is naloxone (Narcan). Choice A is incorrect. The client is in morphine toxicity. The nurse needs to administer an antidote to reverse the symptoms of respiratory depression. Preparing for intubation should not be the nurse's initial action. Choice B is incorrect. The drop in blood pressure is a result of morphine toxicity. Dopamine infusion is not yet necessary as of the moment. Choice D is incorrect. Starting an IV infusion may be necessary; however, in this case, the first action of the nurse would be to administer an antidote to morphine.

The client has just been given an IV dose of morphine 6 mg for neuropathic pain. A few minutes later, the nurse notes that the client's respirations are now 8, and his blood pressure has dropped from 122/83 mmHg to 88/67 mmHg. Which nursing action is the most appropriate? A. Prepare for intubation. B. Prepare to administer a dopamine infusion. C. Administer naloxone. D. Start an IV infusion of normal saline.

B Choice B is correct. If the ICD administers shock, others in physical contact with the patient may feel it but will not be harmed. A shock from the ICD indicates that it's effectively treating the rhythm disorder. Choice A is incorrect. If the ICD administers shock, others in physical contact with the patient may feel it but will not be harmed. Choice C is incorrect. The ICD does not produce any magnetic field. Choice D is incorrect. The shock felt by the patient's friend is an expected outcome of a functional ICD that's treating the patient's rhythm. The question does not mention any clinical symptoms such as chest pain following the shock. This does not necessitate an immediate checkup by a physician. If the patient had just one shock in 24 hours and feels fine after the shock; no immediate intervention is necessary. However, if the patient reports chest pain/ chest pressure/ shortness of breath following an ICD shock or if two or more shocks are experienced within a 24 hour period, the patient should seek medical attention immediately.

The client with an implantable cardioverter defibrillator (ICD) is at the outpatient department. He is concerned about a shock that his friend felt when they were shaking hands. The nurse can discuss which of the following in response to the patient's concern? A. His friend should have an ECG taken to check if his heart rhythm was affected in anyway. B. He shook hands with his friend at the exact same time the ICD delivered a shock to restore his rhythm, and that he need not worry. C. The shock was due to the magnetic field the device emits. D. He should get urgently checked by the physician.

A Choice A is correct. Following a near-drowning incident, the client will most likely exhibit symptoms of hypoxia (decreased oxygen levels in the blood), hypercarbia (increased carbon dioxide levels in the blood), and acidosis (respiratory type) due to a prolonged period of having a lack of oxygen. The introduction of fluid caused this lack of oxygen into the lungs, which disrupted perfusion and gas exchange. Choice B is incorrect. Although the client may be in a coma after near-drowning, hyperthermia and alkalosis are unlikely. There would be a high chance of acquiring hypothermia if the client stayed in the water for too long before being rescued. Alkalosis will not result from a lack of oxygen in the body; instead, acidosis will occur. Although hypothermia is possible in near-drowning situations, lack of oxygen for extended periods will produce hypercapnia/hypercarbia and acidosis. Hypoxia will result from long periods without oxygen, not hyperoxia. Hyperthermia is unlikely to occur in near-drowning incidents. Drowning poses a significant threat to young children and adults who cannot swim. The priority concern associated with drowning is the pulmonary insult caused by the loss of surfactant that impairs gas exchange. An essential treatment for drowning is early cardiopulmonary resuscitation (CPR). Additional therapies include warming measures, stabilizing the client's temperature, and administering prescribed antibiotics to prevent aspirational pneumonia.

The emergency department (ED) nurse is caring for a client who suffered from a near drowning at a local beach. Which of the following assessment findings would the nurse anticipate? A. Hypoxia, hypercarbia, and acidosis B. Coma, hyperthermia, and alkalosis C. Hypothermia, hypocapnia, and alkalosis D. Hyperthermia, hyperoxia, and acidosis

D Choice D is correct. Gentamicin is a weight-based medication requiring the client's pretreatment body weight to calculate the correct dosage. For this reason (and to clarify the above order), it would be reasonable for the nurse to contact the health care provider (HCP) to clarify the above order prior to administration. Regarding the administration of gentamicin, the medication may be given intramuscularly or by intravenous infusion. An additional form of gentamicin is available in eyedrop form. When administering a single dose of intravenous gentamicin, the medication is diluted in 50 to 200 mL of normal saline solution or 5% dextrose in water and intravenously infused over a period of 30 minutes to two hours. Choice A is incorrect. Gentamicin is not recommended to be administered via intravenous push. Choice B is incorrect. Gentamicin is not recommended to be administered via intravenous push. Choice C is incorrect. Gentamicin is not recommended to be administered via intravenous bolus. Peak and trough serum concentrations of gentamicin are required to determine the adequacy and safety of the medication dosage. When such measurements are feasible, they should be carried out periodically during therapy to assure adequate but not excessive drug levels.

The health care provider (HCP) places an order to administer gentamicin intravenously to a client with acute diverticulitis. It is important the nurse knows that intravenous gentamicin is administered: A. Over one minute via IV push B. Over two minutes via IV push C. As an IV infusion over 15-20 minutes D. As an IV infusion over 30 minutes to two hours

D Choice D is correct. The client is exhibiting symptoms and signs of left ventricular heart failure. Decreased cardiac output associated with acute systolic heart failure results in reduced blood pressure, weak pulses, and acute pulmonary edema ( dyspnea, frothy pink sputum, lung crackles). Diltiazem and other calcium channel blockers ( CCBs) produce a negative inotropic effect ( reduced myocardial contractility) and are contraindicated in acute systolic heart failure. CCBs may exacerbate systolic dysfunction and cause heart failure symptoms to worsen. The nurse should question this order to determine if there is a more appropriate medication to accomplish the intended therapeutic effect with a lower risk of complications. Choice A, B, and C are incorrect. These orders are appropriate for a client presenting with acute heart failure and pulmonary edema. The nurse should administer supplemental oxygen to promote adequate tissue oxygenation ( Choice A). Losartan is an angiotensin receptor blocker ( ARV). Afterload is increased in systolic heart failure due to RAAS ( Renin-Angiotensin- Aldosterone-System) mediated increased peripheral vascular resistance. By blocking angiotensin II receptors, losartan causes vasodilation and decreases afterload. Losartan is indicated in systolic heart failure because it reduces mortality and morbidity. Fowler's position ( Choice C) is indicated for patients with heart failure symptoms. Fowler's position promotes oxygenation by allowing maximum chest expansion. Heart failure is associated with reduced cardiac output and reduced blood flow to organs, including the kidneys. Reduced renal blood flow stimulates renin release. Renin converts Angiotensinogen to Angiotensin I, which is further activated to Angiotensin II by the angiotensin-converting enzyme in the lungs. Angiotensin II is a vasoconstrictor, and it increases peripheral vascular resistance (afterload). When medications are used to reduce afterload, the heart pumps better and cardiac output increases. Losartan is an angiotensin receptor blocker ( ARB). It reduces the systemic blood pressure (afterload) by countering the angiotensin II. Losartan does not have direct inotropic action on the heart, but it helps the cardiac output by decreasing the afterload. Losartan improves the morbidity and mortality in heart failure, and hence it's an important drug in treating heart failure.

The nurse attends to a client with shortness of breath, bilateral lung crackles, weak pulses, and frothy pink sputum. Which of the following orders should the nurse question for this client? A. Supplemental oxygen via nasal cannula or mask B. Losartan C. Fowler's position D. Diltiazem

B Choice B is correct. The TNM tumor staging system explores tumor size (T), node involvement (N), and distant metastasis (M). This patient has a small tumor measuring 4.3 cm limited to one portion of the lung, giving it a T staging of T2. Without nodal involvement or metastasis, both N and M are 0. This question is intended to test the representation of N0 and M0 for negative lymph nodes and negative metastases, respectively. The nurse is not required to know "T" staging details. Choice A is incorrect. T3 N3 M1 defines a tumor higher than 7 cm, metastasis in all contralateral lymph nodes, and distant metastasis. Choice C is incorrect. T1 N1 M0 designates a tumor that is self-contained and less than 3 cm in size, has some lymph node involvement, and no metastasis. Choice D is incorrect. T2 N1 M0 implies a tumor that is between 3 and 7 cm in size, has lymph node involvement without distant metastasis.

The nurse evaluating an oncology patient's chart notes that the patient has a tumor in his lung measuring about 4.3 cm in size and accompanying pneumonitis. His cancer does not invade the entire lung and has no metastasis or lymph node involvement. Using the TNM staging system, how would the nurse best classify this patient's tumor? A. T3 N3 M1 B. T2 N0 M0 C. T1 N1 M0 D. T2 N1 M0

A, C, E Choices A, C, and E are correct. Pyelonephritis is an ascending urinary tract infection that involves the kidney. The client exhibits the classic symptoms of cystitis (urinary frequency, dysuria, malaise) along with constitutional symptoms such as fever, chills, and costovertebral tenderness. Choices B and D are incorrect. Jugular venous distention (JVD) is not an expected finding of pyelonephritis as this is a manifestation associated with fluid volume overload. Clients with pyelonephritis typically present with dehydration from the fever and urinary frequency. Urinary frequency is a classic manifestation. Acute pyelonephritis is a complication of cystitis. The infection has spread up the urinary tract and now involves the kidney. Nursing care is like that of cystitis, which includes the administration of prescribed antibiotics, educating the client to stay hydrated, and measures to prevent a recurrence. A complication of pyelonephritis is sepsis. Thus, signs of sepsis, such as tachycardia and hypotension, should be reported to the primary healthcare provider.

The nurse is assessing a client who was just diagnosed with acute pyelonephritis. Which of the following findings should the nurse expect to observe? Select all that apply. A. Costovertebral angle tenderness B. Jugular venous distention C. Fever and chills D. Urinary retention E. Dysuria

A Choice A is correct. Paget's disease is caused by a bone becoming weakened and remodeled, which may result in deformities. The most common area affected by this inappropriate bone remodeling is the skull, pelvis, and spine. Choices B, C, and D are incorrect. Berry aneurysm is an aneurysm that may cause an individual to have a hemorrhagic stroke. This is a common finding for an individual with Polycystic Kidney Disease. Heberden's nodes are a physical feature associated with osteoarthritis. Janeway lesions are an expected finding associated with bacterial endocarditis. These lesions are commonly found on the soles of the feet and the hands. Paget's disease is a disease characterized by accelerated bone remodeling. This may cause an individual to be asymptomatic or have pain in the affected bone(s). The client will be at a higher risk for fracture during this disease process and may eventually develop bone deformities

The nurse is assessing a client with Paget's disease. Which of the following would be an expected finding? A. Bone deformities B. Berry aneurysm C. Heberden's nodes D. Janeway lesions

A, C, E, F Choices A, C, E, and F are correct. A client experiencing opioid withdrawal will experience symptoms such as abdominal cramping, diarrhea, nausea, rhinorrhea, piloerection, diaphoresis, tachycardia, hypertension, insomnia, and agitation. Choices B and D are incorrect. Bradycardia and hypotension are expected findings during opioid intoxication. Opioids are central nervous system (CNS) depressants and may cause life-threatening hypotension, bradycardia, and bradypnea. Opioid withdrawal is typically not life-threatening but highly uncomfortable for the client. Treatment is symptomatic, including encouraging hydration if the client is experiencing vomiting or diarrhea. The nurse should advocate for treatment options to promote abstinence from opioids, such as pharmacotherapy (methadone, naltrexone, etc.) and/or psychotherapy.

The nurse is assessing a client with opioid withdrawal. Which of the following would be an expected finding? Select all that apply. A. Diaphoresis B. Bradycardia C. Irritability D. Hypotension E. Rhinorrhea F. Abdominal cramps

C Choice C is correct. According to Kohlberg's Stages of Moral Development, this pediatric client's behavior correlates to the client being in Level 2 (Conventional Morality Level), Stage 3 (Good Boy-Good Girl) stage of moral development. In this level/stage of moral development, the pediatric client is focused on living up to social expectations and roles. There is an emphasis on conformity, being "nice," and consideration of how choices influence relationships. This knowledge will be beneficial for the nurse to be aware of when caring for this client. Choice A is incorrect. This behavior demonstrates Kohlberg's Level 1 (Pre-conventional Morality Level), Stage 2 (Naively Egotistical) stage of moral development. In this level/stage of moral development, the pediatric client accounts for individual points of view and judges actions based on how they serve individual needs. Pediatric clients at this level (typically ranging from birth to nine years of age) will provide a favor only to receive a favor in return. This client (10 years of age) will be in the Level 2 - Conventional Morality Level. Choice B is incorrect. This behavior demonstrates Kohlberg's Level 2 (Conventional Morality Level), Stage 4 (Law and Social Order) stage of moral development. In this level/stage of moral development, the client realizes the importance of obeying laws, dictums, and social conventions because of their importance in maintaining a functioning society. This client is demonstrating the Stage 3 (Good Boy-Good Girl) stage of moral development, as evidenced by the emphasis on conformity, being "nice," and consideration of how choices influence relationships. Choice D is incorrect. Similar to Choice B (as discussed above), in Stage 4, the client acknowledges the importance of social norms or laws and wants to be a good member of the group or society. This client is not yet in Stage 4. Kohlberg's Stages of Moral Development holds that moral reasoning, the basis for ethical behavior, has six identifiable developmental stages. Kohlberg's six stages were grouped into three levels: pre-conventional, conventional, and post-conventional.

The nurse is caring for a 10-year-old child on the pediatric unit. The nurse, when caring for this age group, should be aware that: A. The child will do something for another person if that person does something for the child. B. The child now follows social standards for the good of all. C. The child wants to follow the rules because of a need to be seen as "good." D. The child finds satisfaction in following rules.

A Choice A is correct. Administering the IV antibiotic is the top priority in a client with cystic fibrosis (CF) that develops a fever. Due to the excessively thick mucus that builds up in their bronchi and bronchioles, children with CF are incredibly susceptible to respiratory infections. A fever is an indication of infection and aggressive management is the top priority. Choice B is incorrect. Pancreatic enzymes are administered to children with CF within 30 minutes of any meal and snack. These are given to aid in digestion since the excessive, sticky mucus clogs up the pancreatic duct in these clients. This is a standard medication given every day, but is not the top priority when a child with CF develops a fever. Choice C is incorrect. Fat soluble vitamins are a daily medication for children with CF. Due to the buildup of excessive, sticky mucus in their bile duct, children with CF do not absorb fat normally. This leads to a deficiency in fat soluble vitamins, which are vitamins A, D, E, and K. This is a standard medication given every day, but is not the top priority when this client develops a fever. Choice D is incorrect. Albuterol is a bronchodilator frequently given as a nebulizer treatment to clients with CF. Although this medication might be given top priority if the client was experiencing respiratory difficulty, the question states they have developed a fever. Due to this finding, the IV antibiotics are the top priority as CF clients are very susceptible to infections.

The nurse is caring for a 16-year-old client with cystic fibrosis. The client develops a temperature of 101.2 degrees F (38.4C). Which medication does the nurse administer with top priority? A. IV antibiotic B. Pancreatic enzyme C. Fat soluble vitamin D. Albuterol

B Choice B is correct. Typically, healthy kidneys excrete 80-90% of the body's potassium. When there is injury or damage to the kidneys, such as with acute renal failure, potassium excretion is impaired. Metabolic acidosis can also occur because of the decreased ability to filter acids and reabsorb bicarbonate. Hence, as hydrogen ions enter the cells, potassium is pushed out of the cells and into the extracellular fluid. If the acute renal failure is related to trauma, the damaged cells release additional potassium into the extracellular fluid. These processes all increase the body's potassium, so the client would be at risk of developing high potassium levels (hyperkalemia). Choice A is incorrect. Cushing syndrome puts a client at risk for hypokalemia, not hyperkalemia. These clients experience decreased potassium levels due to increased urinary losses of potassium and excess cortisol production, which interferes with the sodium/potassium pump action. Choice C is incorrect. Cystic fibrosis is an endocrine disease that impacts the function of multiple organs. It puts clients at risk of low potassium levels (hypokalemia) due to renal potassium wasting, increased sweating, and metabolic acidosis. Choice D is incorrect. Bulimia nervosa is an eating disorder characterized by periods of binge eating, followed by inappropriate, extreme weight control methods. Repeated vomiting, diuretic use, and/or laxative use all deplete the body's potassium stores, so this client would be at risk for hypokalemia, not hyperkalemia.

The nurse is caring for a client on a medical floor. The nurse would recognize that which diagnosis increases the client's risk of developing hyperkalemia? A. Cushing's syndrome B. Acute renal failure C. Cystic fibrosis D. Bulimia nervosa

B Choice B is correct. Mechanical ventilation may cause a stress ulcer. A proton pump inhibitor (PPI) or a histamine-2 receptor antagonist (H2 blocker) may be utilized to prevent this ulcer which may lead to a gastrointestinal bleed. Choice A, C, and D are incorrect. Hydroxyzine is an anticholinergic utilized in allergic reactions and anxiety. Rivastigmine increases acetylcholine in the central nervous system and is indicated for dementia. Verapamil is a calcium channel blocker indicated for hypertension and migraine headache prophylaxis. These medications have no relevance to mechanical ventilation management. Mechanical ventilation poses a risk for a stress ulcer to form. This is caused by hypersecretion of gastric acid and impaired protection of the gastric mucosa. Stress ulcers pose a serious risk as they may cause gastrointestinal bleeding. This bleeding may lead to perforation and then shock. PPIs such as pantoprazole or H2 blockers famotidine may be used to mitigate this risk. Manifestations of stress ulcers include hematemesis, melena, anemia, and shock.

The nurse is caring for a client receiving mechanical ventilation. Which prescription from the primary healthcare physician (PHCP) should the nurse anticipate? A. Hydroxyzine B. Pantoprazole C. Rivastigmine D. Verapamil

C Choice C is correct. Elevating the residual limb using a pillow is an incorrect intervention, therefore the correct answer to this question. Proper positioning of the residual limb is crucial in preventing flexion contractures. For the first 24 hours, the residual limb should be elevated using a pillow to increase venous return and decrease edema. However, beyond 24 hours, the pillow must be removed and the residual limb should be placed flat on the bed. Elevation of the residual limb on a pillow beyond 24 hours makes the client with above-knee-amputation prone to hip flexion contractures. A flexion contracture refers to the shortening of muscles and tendons leading to deformity and rigidity of joints. The client should be encouraged to lay prone for at least 30 minutes several times a day to reduce the risk of contractures. Prolonged sitting in a chair and semi-Fowler's position must be discouraged. The nurse should also educate the client to avoid external rotation of the hip by using a trochanter roll in bed. Choice A is incorrect. This is a correct intervention. Placing the client in a prone position stretches the muscles and helps prevent hip flexion contractures. Choice B is incorrect. This is a correct intervention. Elevating the foot end of the bed helps prevent edema; keeping the head flat helps in preventing hip flexion contractures. Choice D is incorrect. This is a correct intervention. Elastic wraps on the client's residual limb help with swelling, minimizing pain, and molding it in preparation for a prosthesis.

The nurse is caring for a client that underwent an above-the-knee amputation more than 24 hrs ago. Which intervention by the nurse should not be included in the care of the client? A. Placing the client in prone position. B. Elevate the foot of the bed with the head flat. C. Elevate the residual limb on a pillow. D. Maintain the application of an elastic compression wrap.

A Choice A is correct. Classic manifestations associated with a vaso-occlusive crisis include diffuse pain and signs of dehydration. Choices B, C, and D are incorrect. Increased WBCs is a classic finding for a client experiencing a VOE. This is caused by the inflammation occurring with this disease process. Fatigue is a common complaint associated with the disease process overall. This is not specific to a VOE which is manifested by hypovolemia and pain. Increased skin tenting in the sternum would be evidence of dehydration and would indicate the client is not meeting the treatment goals, which involves repleting fluids. Sickle cell disease (SCD) is a genetic disorder that causes abnormalities in hemoglobin. The client has an increased presence of hemoglobin S compared to a decreased hemoglobin A. Vaso-occlusive event (VOE) may be triggered by a client having a decrease in fluid, changes in altitude, stress, infection, or use of substances. Nursing care aims to provide appropriate hydration with a combination of hypo- or isotonic fluids, optimal pain control with opioids, and oxygen as clinically indicated. To promote comfort, the nurse should remove constrictive clothing and keep the room temperature warm.

The nurse is caring for a client who has a vaso-occlusive event (VOE) secondary to sickle cell disease (SCD). Which of the following would indicate the client is achieving the treatment goals? A. Decreased self-report of pain. B. Increased white blood cell (WBC) count. C. Decreased energy while ambulating. D. Increased skin tenting in the sternum.

A Choice A is correct. Diphenhydramine is an anticholinergic and is utilized for dystonic reactions associated with antipsychotic use (such as fluphenazine, a typical antipsychotic). Dystonia is one of the earliest adverse effects and should be promptly reported to the prescriber. Choices B, C, and D are incorrect. Mannitol is an osmotic diuretic indicated for increased intracranial pressure. This medication would not be used for dystonic reactions. Thiamin is a B-vitamin and can be helpful for alcohol withdrawal. This is not indicated for the treatment of dystonia. Haloperidol is a typical antipsychotic and would be detrimental in treating dystonia. Medications like fluphenazine include haloperidol which would worsen the effect. Acute dystonia is a sudden, sustained contraction of one or several muscle groups, usually of the head and neck. Acute dystonias can be frightening and uncomfortable, but they are not dangerous unless they involve muscles affecting the airway, which is rare. However, they cause significant anxiety and should be treated promptly with anticholinergics such as diphenhydramine. Dystonia is caused by medications blocking dopamine. Classically, this involves antipsychotic medications (especially the typicals) and metoclopramide. Typical antipsychotics include fluphenazine, haloperidol, and chlorpromazine

The nurse is caring for a client who has developed dystonia following the administration of fluphenazine. Which medication does the nurse anticipate that the primary healthcare provider (PHCP) will prescribe? A. diphenhydramine B. mannitol C. thiamine D. haloperidol

B Choice B is correct. Fondaparinux is a selective inhibitor of factor Xa, which is indicated for prophylaxis or treatment of DVT or PE. This medication is given subcutaneously once daily. Choices A, C, and D are incorrect. Fondaparinux is not indicated in treating sickle cell anemia, hemophilia, or pernicious anemia. Hemophilia is a clotting disorder (with a lack or absence of factor VIII or IX) and requires treatment. Anticoagulants should not be used for clients with hemophilia because this would increase their risk of bleeding. Pernicious anemia requires vitamin B12 as a treatment, not an anticoagulant. Sickle cell anemia is a hereditary condition that causes increased levels of hemoglobin-S. ✓ Key monitoring parameters for a client receiving fondaparinux include renal function as prolonged exposure may be harmful and reflect increased creatinine. ✓ A dose adjustment (or a new anticoagulant) will be required for a client with renal impairment. ✓ Monitoring of the aPTT and the platelet count is not necessary with this medication. ✓ This medication is given subcutaneously once daily.

The nurse is caring for a client who has newly prescribed fondaparinux. The nurse understands that this medication is intended to treat which condition? A. Hemophilia B. Venous thromboembolism C. Sickle Cell Anemia D. Pernicious Anemia

A, B, D, E Choices A, B, D, and E are correct. Nausea and vomiting are common during the first trimester because of increased human chorionic gonadotropin and decreased gastric motility. Non-pharmacological measures include instructing the client to eat dry crackers before getting out of bed in the morning, consume fluids at least 30 minutes before or after solid food, brush their teeth immediately after a meal to decrease the aftertaste of food, and avoid overfilling the stomach which can be irritating and trigger nausea and vomiting. Choice C is incorrect. Lying down after a meal will decrease gastric motility. Decreasing gastric motility will allow the food and fluid to remain in the stomach, making nausea and vomiting more likely. The client should be upright to promote gastric emptying. ✓ Morning sickness is not harmful to the fetus and usually resolves by week 12. ✓ Treatment goals include preventing maternal dehydration, reducing symptoms, and correcting vitamin deficiencies. ✓ Nonpharmacological treatment involves trigger avoidance (foods with strong smells/aromas, snacks before getting out of bed, avoidance of overfilling the stomach, and consume fluids at least 30 minutes before or after solid food to minimize the effect of a full stomach. ✓ Pharmacotherapy may be pyridoxine (vitamin B6) or doxylamine/pyridoxine.

The nurse is caring for a client who is experiencing nausea associated with her pregnancy. The nurse should recommend that the client Select all that apply. A. eat dry crackers before getting out of bed in the morning. B. consume fluids at least 30 minutes before or after solid food. C. lie down soon after eating. D. brush their teeth immediately after a meal. E. avoid overfilling your stomach.

A, B, E Choices A, B, and E are correct. A is correct. The normal serum calcium level is 9-10.5 mg/dL. This client has a high serum calcium level (hypercalcemia). Phosphorus is a medication the nurse would expect to administer to treat hypercalcemia. Phosphorus and calcium have an inverse relationship, so by increasing the serum level of phosphorus the nurse can decrease the serum level of calcium. Oral phosphate is the preferred method of administering phosphorus. If given IV, calcium phosphate forms and precipitates in the tissues. This precipitation phenomenon reduces serum calcium levels very quickly. B is correct. Calcitonin is a medication the nurse would expect to administer to treat hypercalcemia. Calcitonin is a thyroid hormone that decreases the plasma calcium level by inhibiting bone resorption and lowering the serum calcium concentration. E is correct. Bisphosphonates are intravenous osteoporosis drugs that can quickly lower calcium levels and are often used to treat hypercalcemia due to cancer. Choice C is incorrect. Vitamin D should be avoided in hypercalcemia. Vitamin D enhances the absorption of calcium and can therefore increase the level of serum calcium, which we do not want to do when the client's level is already high. Choice D is incorrect. IV calcium gluconate is given to patients that are hypocalcemic, not hypercalcemic. It can treat the tetany that occurs when a client is severely hypocalcemic. It can also be given to protect the cardiac muscle if a client has severe hyperkalemia or hypermagnesemia.

The nurse is caring for a client whose latest lab results show a serum calcium level of 13.2 mg/dL. Which medication does the nurse expect to administer based on this lab result? Select all that apply. A. Phosphorus B. Calcitonin C. Vitamin D D. IV calcium gluconate E. IV Bisphosphonates

A, C, D Choices A, C, and D are correct. Individuals with Borderline Personality Disorder (BPD) often engage in self-harm/parasuicide behaviors in which the intent is not death. These gestures may be superficial cutting, etc. All clients should be assessed for suicide regardless of their diagnosis. Therapeutic boundaries should be established as a characteristic of this personality disorder is polarizing individuals and splitting. Referring the client for group therapy is one of the cornerstone treatments for BPD. Choices B and E are incorrect. Independent decision-making is not impaired for an individual with BPD. This would be an intervention for Dependent Personality Disorder. Finally, the client with BPD can establish social relationships - although they may be unstable, this would be an intervention for Avoidant Personality Disorder. Borderline Personality Disorder occurs more in females than males and has features such as self-harm/parasuicidal behavior, splitting, unstable relationships, an unclear self-image, and impulsivity.

The nurse is caring for a client with Borderline Personality Disorder. Which of the following actions should the nurse take? Select all that apply. A. Assess the client for suicidal ideation B. Encourage independent decision-making C. Establish therapeutic boundaries D. Refer the client for group therapy E. Encourage social relationships

D Choice D is correct. Schizophrenia is treated with antipsychotic medications. Typical (or first-generation) antipsychotic drugs include haloperidol, fluphenazine, and chlorpromazine. Atypical (second generation) antipsychotic medications include quetiapine, ziprasidone, and risperidone. Choices A, B, and C are incorrect. Lithium is indicated for the treatment of Bipolar disorder. Bupropion is an atypical antidepressant indicated in major depressive disorder. Sertraline is a selective serotonin reuptake inhibitor and is indicated for major depressive and anxiety disorders. Schizophrenia is a psychotic disorder characterized by positive (hallucinations) and negative symptoms (lack of motivation). Most cases of schizophrenia have an onset in adolescence. Acute stabilization and maintenance treatment is accomplished by prescribed antipsychotic medications such as risperidone, haloperidol, or fluphenazine.

The nurse is caring for a client with schizophrenia. The nurse should anticipate a prescription for which medication? A. Lithium B. Bupropion C. Sertraline D. Risperidone

C Choice C is correct. While it is unlikely that this child will have cystic fibrosis, genetic testing can provide more information. Carrier testing may be recommended for people with a family history of this genetic disorder. Cystic fibrosis is an inherited autosomal recessive disorder that only appears in an infant whose parents have the mutated gene. Choice A is incorrect. There is not enough evidence to tell the patient that her child is at high risk for developing cystic fibrosis. Choice B is incorrect. There is not enough information to say that this child will not be born with cystic fibrosis. Choice D is incorrect. Her diet is irrelevant in regards to whether or not she will pass cystic fibrosis to her child.

The nurse is caring for a married 31-year-old second-time pregnant female. Her first child is 4-years-old who was born vaginally and is considered to be a healthy preschooler. The patient explains that she's worried that her newborn child will be born with cystic fibrosis because her sister's daughter has it. What information about this genetic condition should the health care provider share? A. Her child is at an extremely high risk for passing cystic fibrosis to her child. B. Her child is not at risk for developing cystic fibrosis. C. While it is unlikely that this child will be born with cystic fibrosis, genetic testing may relieve some of the distress. D. She needs to adjust her diet in order to prevent passing on the medical condition.

B Choice B is correct. Addressing the client's active bleeding is the priority, as the client may develop shock if it goes untreated. Bleeding associated with hemophilia is often found in the joint space or at the point of physical injury. Choices A, C, and D are incorrect. Acute pain may be associated with a bleeding episode, but the pain is often brief and is not life-threatening. Chronic illnesses may negatively impact self-esteem, and the nurse should focus only on this aspect once the physical needs have been met. Enhancing the adolescents' knowledge to avoid contact sports is important, but this is not an immediate physical need compared to bleeding.

The nurse is caring for an adolescent with hemophilia who reports pain and joint bleeding after playing baseball. The nurse should prioritize the client's A. acute pain. B. bleeding. C. self-esteem. D. knowledge deficit.

A Choice A is correct. Following a femoral angioplasty, the affected extremity should be assessed for a pulse, and the client should be instructed to report any decreased sensation. A common complication following this procedure is arterial occlusion which causes a decreased pulse and the client to experience a reduced sensation (or paresthesias). Choices B, C, and D are incorrect. Any client who refuses a medication should receive follow-up as the nurse is instrumental in explaining the purpose of the medication. However, this is not the priority because the client would adversely face hyperglycemia which is not immediately life-threatening. A GCS of 14 (with the highest score of 15) is optimal and does not require immediate follow-up. Finally, bowel and bladder function disturbances are commonly seen with a thoracic spinal cord injury. However, this is not of immediate concern, considering the last bowl movement was one day ago. The neurovascular assessment is a priority following a femoral angioplasty. This includes assessing the affected extremities' distal pulse and temperature. Additionally, the client should report any signs of decreased sensation or paresthesia. Antiplatelets are commonly prescribed following this procedure to prevent occlusion.

The nurse is caring for assigned clients. The nurse should initially assess the client who A. is recovering from a femoral angioplasty and reports their foot as falling asleep. B. has diabetes mellitus and refused their prescribed glargine insulin. C. received alteplase three hours ago for a stroke and has a Glasgow Coma Scale of 14. D. had a T6 spinal cord injury and has not had a bowel movement since yesterday.

C Choice C is correct. The client yelling and shouting at other clients requires immediate intervention because this situation is hostile and warrants the nurse to de-escalate the situation before it intensifies. Choices A, B, and D are incorrect. A client repeatedly washing their hands is a feature of obsessive-compulsive disorder, and the nurse should not intervene unless the act threatens the client or others. Further, a client talking over others in therapy will require intervention, but this is not the immediate need as it is not a hostile situation. Finally, voluntarily admitted clients might request discharge, but this is a low-priority item compared to the client yelling at others. When prioritizing client needs, focus on ensuring that physiological, safety and security needs are met first. In this question, the client's safety and security needs are prioritized over the other needs.

The nurse is caring for the following assigned clients. The nurse should initially follow-up with the client who A. is repeatedly washing their hands. B. talking over others during group therapy. C. yelling and shouting at others. D. is voluntarily admitted and requesting discharge.

A,B,C Choices A, B, and C are correct. According to the Centers for Disease Control (CDC), risk factors for TB include immunosuppression, organ transplant, chronic corticosteroid use, substance use, diabetes mellitus, and residing in environments such as nursing homes, prisons, and homeless shelters. Choices D and E are incorrect. Obesity is not a risk factor for TB. Low body weight is a risk factor for TB. Influenza vaccination is irrelevant to pulmonary tuberculosis and does not raise or lessen the risk for infection.Pulmonary tuberculosis is a pathogen spread via aerosolized droplets ➢ If TB is suspected, the client should be placed in airborne precautions. This includes a room with negative pressure or a specialized HEPA filter. The client should not be cohorted with another client and the door must remain closed. ➢ Signs and symptoms of TB include pleuritic chest pain, fever, chills, night sweats, lethargy, and bloody sputum. ➢ When an individual is symptomatic with TB, they have a high risk of transmitting it to others. ➢ Once the symptoms recede, the risk of transmission significantly decreases.

The nurse is conducting a health screening at a local health fair. Which of the following should the nurse recognize as a risk factor for developing pulmonary tuberculosis (TB)? Select all that apply. A. Human Immunodeficiency Virus (HIV) B. Organ transplant C. Chronic corticosteroid use D. Influenza vaccination E. Obesity

B Choice B is correct. Homonymous hemianopia (HH) is vision loss on the same side of the visual field in both eyes. It is appropriate for the nurse to teach the client to scan the room. Scanning the room will expand the visual field because the same half of each eye is affected. Choices A, C, and D are incorrect. An eye patch is an appropriate intervention for a client with double vision (diplopia). HH is not a problem with hearing and changing the approach to speaking to a client and providing ear plugs is irrelevant to this disorder. Homonymous Hemianopia is characterized by vision loss on the same side of the visual field in both eyes. This is usually caused by a stroke, tumors, or epilepsy. Visual field loss is indicative of a lesion involving the visual pathway posterior to the chiasm.

The nurse is planning care for a client with homonymous hemianopia. The nurse should plan for which intervention in the care plan? A. Place an eye patch over the affected eye B. Instruct the client to turn their head from side to side C. Speak slowly, clearly, and in a deeper voice D. Provide the client with ear plugs to promote rest

13 units

The nurse is preparing to administer a regular insulin IV bolus to a client who has hyperglycemic-hyperosmolar state (HHS). The primary health care provider (PHCP) has prescribed an initial bolus dose of 0.1 unit/kg. The client weighs 277 lbs. How much regular insulin should the nurse administer to the client as an IV bolus? Fill in the blank. Round your answer to the nearest whole number.

C, D Choices C and D are correct. Lisinopril is an ACE inhibitor (ACE-I) and may cause hyperkalemia. It would be correct for the nurse to instruct the client to limit their intake of potassium-rich foods such as avocados, bananas, apricots, and legumes. The client decreasing their potassium intake could decrease the likelihood of developing dangerously high potassium. ACE-I's may also cause a client to have decreased taste sensation. This may make the client more likely to use salt, worsening hypertension. Thus, it is appropriate to instruct the client to season their food more naturally. Choices A, B, and E are incorrect. ACE-Is effectively lowered blood pressure and reduced congestive heart failure mortality. These medications do not lower pulse, and therefore, it would be inappropriate for the client to take their pulse prior to taking the medication. ACE-I do not have a diuretic effect and it would be inaccurate to advise the client that they may need to use the bathroom more frequently. Finally, ACE-I lower blood pressure, not cholesterol. ACE inhibitors are central in the treatment of heart failure and hypertension. These medications inhibit the deleterious effects associated with Angiotensin II. Medications in this class include lisinopril, enalapril, and captopril. The most serious adverse effect is angioedema. A nagging, dry cough is a common side-effect associated with this medication, and if this should occur, a prescriber may switch the client to an ARB such as valsartan. Finally, these drugs may raise potassium and creatinine.

The nurse is providing discharge instructions to a client prescribed lisinopril. Which of the following instructions should the nurse include? Select all that apply. A. You will need to take your pulse for one minute before each dose. B. You may notice the need to go to the bathroom more often. C. Limit your intake of foods such as avocados and apricots. D. You may notice a decrease in your ability to taste foods. E. The goal of this medication is to lower your cholesterol.

B, C, E Choices B, C, and E are correct. Garlic is a food rich in phosphorus and would be an appropriate recommendation for a client that needs to incorporate more phosphorus in their diet. Many nuts are rich in phosphorus and are an excellent way to increase the dietary intake of this important mineral. Cashews, almonds, and brazil nuts are all very high in phosphorus. One cup (140 grams) of roasted turkey contains around 300 mg of phosphorus, more than 40% of the recommended daily intake (RDI). Choices A and D are incorrect. While leafy greens are good for many vitamins and minerals, they do not contain a lot of phosphorus. Therefore, this would not be a good choice to recommend to a client that needs a diet rich in phosphorus. Butter does not have a lot of phosphorus. This would not be an appropriate recommendation.

The nurse is reviewing teaching with a client who has been advised to eat foods rich in phosphorus. What foods should the nurse include in dietary teaching with the client that are good sources of phosphorus? Select all that apply. A. Leafy greens B. Garlic C. Nuts D. Butter E. Turkey

C Choice C is correct. Celiac disease requires standard precautions. It is not an infectious disease and is not transmitted from person to person; therefore, there is no reason to initiate any additional precautions. Choice A is incorrect. Droplet precautions are not indicated for the patient with Celiac disease. It is not an infectious disease and is not transmitted from person to person. Droplet precautions are indicated when there is an infection that can spread by speaking, sneezing, or coughing near someone. Examples include influenza, adenovirus, and rhinovirus. Choice B is incorrect. Contact precautions are not indicated for the patient with Celiac disease. It is not an infectious disease and is not transmitted from person to person. Contact precautions are indicated when there is an infection that can spread by touching the patient or items in the room. Examples include MRSA and VRE. Choice D is incorrect. Neutropenic precautions are not indicated for the patient with Celiac disease. Neutropenic precautions are indicated when the patient is immunocompromised and at additional risk for infection. Examples of patients on neutropenic precautions may be those who have received a transplant, or who have a low absolute neutrophil count due to cancer or chemotherapy.

The nurse is setting up the room for a patient newly diagnosed with Celiac disease. She knows to place the patient on which of the following precautions? A. Droplet precautions B. Contact precautions C. Standard precautions D. Neutropenic precautions

B Choice B is correct. A CST is indicated for high-risk clients who are in the third trimester. CST requires the client to have contractions either through oxytocin administration or nipple stimulation. Choices A, C, and D are incorrect. The client consuming a liquid with concentrated glucose is not indicated for a CST. This is appropriate for a glucose tolerance test. The results for a CST are interpreted as follows - Positive (abnormal) indicates that late decelerations were present in the FHR in more than 50% of the contractions. Negative (normal) indicates that no late or variable decelerations were evident during the contractions. A contraction stress test is indicated for clients with high-risk pregnancies. This test is completed in the third trimester. Relative contraindications to a CST include placenta previa and patients at risk for preterm delivery as this test may hasten delivery or at minimal cause rupture of the membranes.

The nurse is teaching a client about a scheduled contraction stress test (CST). Which of the following statements should the nurse include? A. "You will need to consume a liquid with 50 grams of glucose." B. "You may need to stimulate your nipples during this test." C. "A positive result means your baby has had no late decelerations." D. "A negative result means your baby has had variable decelerations."

A, B, C Choices A, B, and C are correct. Rho(D) Immune Globulin should be administered to Rh-negative women who have been exposed to Rh-positive blood. Such exposures may be linked to Delivering an Rh-positive infant Chorionic villus sampling Aborting an Rh-positive fetus Receiving accidental transfusion of Rh-positive blood Amniocentesis Intraabdominal trauma while carrying an Rh-positive fetus. Choices D and E are incorrect. Transvaginal ultrasound is not invasive and does not risk mixing maternal and fetal blood. Therefore, transabdominal ultrasound is not an indication to administer Rho(D) Immune Globulin. Rho(D) Immune Globulin is not indicated for noninvasive antepartum testing, such as a non-stress test (NST). Rh isoimmunization (Rh alloimmunization/ Rh sensitization) is a phenomenon in which an Rh-negative mother develops anti-D antibodies against Rh-positive fetal RBCs. In these cases, the Rh antigen on the fetal RBCs is acquired from paternal inheritance. Fetomaternal hemorrhage during a prior pregnancy, trauma, abortion, or any invasive procedure (e.g., chorionic villus sampling, amniocentesis) may expose an Rh negative maternal circulation to Rh positive fetal RBCs resulting in the development of antibodies. These antibodies do not affect the firstborn child. However, subsequent newborns can develop hemolytic disease. In at-risk mothers, isoimmunization can be prevented by administering Rh immune globulin or anti-D immunoglobulin (RhoGAM). RhoGAM prevents the production of anti-Rho(D) antibodies in Rh-negative women who have been exposed to Rh-positive blood. RhoGAM acts by suppressing the immune reaction of the Rh-negative woman to the antigen in Rh-positive blood, preventing antibody response and thereby preventing hemolytic disease of the newborn in future Rh-positive pregnancies. A failure to administer or delay in giving RhoGAM can destroy fetal RBCs, leading to hemolytic disease in the newborn. Important points to note before administering RhoGAM:- Type and antibody screening of the mother's blood and cord blood type of the newborn should be performed to determine the need for the medication. The mother must be Rh-negative and negative for Rh antibodies. The newborn must be Rh-positive. In the event of termination of pregnancy, if the fetal blood type is uncertain, RhoGAM should be administered to an Rh-negative mother. The newborn might have a weakly positive antibody test if the woman received Rho(D) immune globulin during pregnancy. The drug is administered to the mother, not the infant. The deltoid muscle is recommended for intramuscular administration. The medication may be given intravenously if prescribed.

The nurse is teaching a group of students about Rho(D) Immune Globulin. It would be correct if the student states that this medication is indicated when? Select all that apply. A. delivering an Rh-positive infant. B. aborting an Rh-positive fetus. C. undergoing chorionic villus sampling. D. having a transvaginal ultrasound. E. non-stress testing (NST).

C, E Choices C and E are correct. Hypokalemia is when the potassium level is less than 3.5 mEq/l. Conditions such as metabolic alkalosis, potassium wasting diuretics, Cushing's syndrome, and alcoholism may all contribute to hypokalemia. Choices A, B, and D are incorrect. Hyperkalemia is when the potassium level is above 5.0 mEq/l. Conditions causing this include diabetic ketoacidosis, chronic renal failure, and Addison's disease. While a client in DKA is dehydrated, they are in an acidotic state, increasing their potassium level. The normal potassium level is 3.5-5.0 mEq/l. Conditions causing hypokalemia include metabolic alkalosis, which causes an intracellular shift of potassium. Cushing's syndrome or disease causes hypokalemia because of this increase in aldosterone (potassium elimination and sodium retention). Symptoms of hypokalemia include muscle weakness, cramping, and lethargy. Cardiac rhythm changes include flattened T-waves and the presence of U waves.

The nurse is teaching a group of students the causes of hypokalemia. It would indicate a correct understanding of the student if they stated which condition causes this electrolyte imbalance? Select all that apply. A. Diabetic ketoacidosis B. Addison's disease C. Metabolic alkalosis D. Chronic renal failure E. Cushing's syndrome

D Choice D is correct. Hormonal implants may be placed subdermally and should be removed, and if the client elects, replaced after three years. Three years is the approved duration for this device. Choices A, B, and C are incorrect. IUDs may cause serious maternal and fetal consequences, including maternal infection, miscarriage, and placental abruption. If the client is pregnant, the IUD most likely needs to be discontinued. Contraceptive rings that are expelled or inadvertently removed may be reinserted as long as they are rinsed with cool water and within two to three hours. Combined estrogen-progestin contraception may adversely cause hypertension. The etonogestrel contraceptive implant is advantageous because it allows the client to attain effective contraception without having to take daily medication. The insertion site is typically on the upper arm overlying the triceps muscle. This contraceptive device's most common adverse effects include headache, weight gain, acne, and breast tenderness. This device is approved to stay in place for three years and may be inserted in an outpatient setting.

The nurse is teaching individuals at a local health fair about female contraception options. Which information should the nurse include? A. "Intrauterine devices (IUDs) may be safely continued during pregnancy." B. "Contraceptive rings may be rinsed with rubbing alcohol if they are expelled." C. "Combined estrogen-progestin contraception may lower your blood pressure." D. "Hormonal implants that are placed subdermally need to be removed after 3 years."

D Choice D is correct. It is most appropriate to request an interpreter from the hospital's interpreter service. A certified medical interpreter has the proper training to quickly and accurately translate the conversation as well as protect client confidentiality. This is the appropriate action by the nurse. Choice A is incorrect. It is not appropriate to ask around to see if anyone nearby knows Spanish. This person would not be a certified medical interpreter and have the necessary training to properly interpret the conversation. Choice B is incorrect. It is not appropriate to call the receptionist who speaks Spanish to translate. This does not respect patient privacy, nor does the receptionist have the necessary training to properly interpret the conversation like a certified medical interpreter does. Choice C is incorrect. It is not appropriate to pull up Google translate on the internet to interpret the conversation. Not only would this be inaccurate, but it would take far too long to figure out what the patient needs. This is not an acceptable action.

The nurse is working at the triage desk in the emergency department when a client arrives and begins speaking in Spanish. The nurse asks if the client would like an interpreter, and the client says, "No." What is the most appropriate action for the nurse to take? A. Ask around to see if anyone nearby knows Spanish. B. Call the receptionist who speaks Spanish to translate. C. Pull up Google translate on the internet. D. Request an interpreter from the hospital's interpreter service despite the client's refusal. This is the safest option and will lead to the best outcome for the client.

D Choice D is correct. Signs and symptoms of heat exhaustion include headache, dizziness, nausea, and weakness. Due to excessive sweating, hyponatremia can also be present. Choice A is incorrect. Although profuse sweating is one sign of myocardial infarction, all signs and symptoms point to heat exhaustion/heat stroke since the client has been out in the open exposed to the sun. There is no complaint of chest pain from the client which could indicate a myocardial infarction. Choice B is incorrect. All signs and symptoms point to heat exhaustion/heat stroke since the client has been out in the open exposed to the sun. Signs of heart failure include respiratory symptoms from pulmonary edema, which include cough, crackles, and shortness of breath. All of which was not manifested by this client. Choice C is incorrect. Signs and symptoms of pulmonary embolism are tachypnea, dyspnea, anxiety, and chest pain. The client did not display any of these signs or symptoms.

The nurse is working in the emergency department when an elderly man is rushed in with complaints of dizziness, weakness, headache, and nausea. The patient's shirt is drenched in sweat. His son who accompanied him to the hospital tells the nurse that they were watching his 10-year-old grandson's baseball game when suddenly his dad complained of a severe headache and weakness. The nurse understands that based on the client's presentation and history, he is most likely suffering from: A. Myocardial infarction B. Left-sided heart failure C. Pulmonary embolism D. Heat exhaustion

C Choice C is correct. If the prenatal client has a current case of placenta previa, the cervix should not be assessed for dilation. Placenta previa arises when the placenta develops in a problematic spot, close to or over the cervical os. To prevent bleeding or premature labor, women with placenta previa shouldn't have their cervix checked manually. Instead, an ultrasound may be performed. Choice A is incorrect. Ultrasounds may be used safely in women with placenta previa. Ultrasounds are the safest way to assess cervical dilation in a woman with this issue. Choice B is incorrect. If the physician requires, the nurse may safely palpate the abdomen and thus the uterus of a woman whose pregnancy is difficult because of placenta previa. Choice D is incorrect. Laying on her left side is an often-used position for pregnant women, including those with placenta previa. This position increases circulation to the fetus and is often a comfortable position for laboring women.

The nurse notes that the 39-week pregnant client is experiencing placenta previa. Knowing the contexts surrounding this condition, the nurse refrains from performing which of the following standard procedures? A. Ultrasonography of the uterus B. Palpating the uterus to determine fetal arrangement C. Checking the cervix for dilation D. Placing the patient on the left side

A Choice A is correct. Obtaining a throat culture would require immediate follow-up because this may cause acute laryngospasm leading to respiratory obstruction. The culture may be obtained once an artificial airway has been established. Choices B, C, and D are incorrect. These actions are appropriate during acute epiglottitis. High Fowler's position would assist with respiration. Peripheral vascular access is necessary to administer broad-spectrum antibiotics and corticosteroids. Airway equipment such as a bag-valve mask should be kept at the bedside in the event the client requires intubation. This may be used to oxygenate the client before intubation temporarily. Epiglottis is a cartilaginous flap present at the back of the throat. It's primary function is to close over the airway during swallowing so that the food does not enter the airway. Acute epiglottitis is a medical emergency that has an abrupt onset. In epiglottitis, the epiglottis becomes inflamed and swollen and constructs the airway. Classic symptoms of epiglottis include - Sore throat and pain in swallowing Fever The child insists on sitting upright and leaning forward (tripod position), with the chin thrust out, mouth open, and tongue protruding. Drooling of saliva Red and inflamed mucous membranes Large, cherry red, edematous epiglottis Prevention : Key prevention for epiglottitis is immunization with H. influenzae type B conjugate beginning at two months of age.

The nurse preceptor supervises a new nurse caring for a child with epiglottitis. Which action by the new nurse would require the nurse preceptor to intervene? The new nurse A. obtains a throat culture. B. positions the client high-Fowlers. C. initiates peripheral vascular access. D. places a bag-valve mask at the bedside.

A Choice A is correct. This is an example of transference. In transference, the client's unconscious feelings toward a healthcare worker come to the surface that originally stems from someone else. For instance, if a client starts to have hostility towards the healthcare worker because they remind them of a family member with whom they had (or have) a negative relationship. The client bringing up their mother and pinning it on the healthcare worker exemplifies transference. Choices B, C, and D are incorrect. Instead of breaking objects, the client joining a kickboxing class demonstrates sublimination, a positive defense mechanism. The client denying their alcoholism is an example of denial, which is a common defense mechanism used in borderline personality disorder. The client stating she will kill herself if her boyfriend leaves her is an example of manipulation. This is commonly used in borderline personality disorder. Transference refers to the client's unconscious feelings toward a healthcare worker originally felt in childhood for a significant other. The client may say something like, "You remind me exactly of my brother." Countertransference refers to unconscious feelings that the healthcare worker has toward the client. For instance, if the client reminds you of someone you do not like, you may unconsciously react as if the client were that individual.

The nurse supervises a novice nurse interviewing a client with a borderline personality disorder. Which client statement would demonstrate the client using transference? A. "You are just like my mother bothering me with these questions." B. "Instead of breaking objects, I have joined a kickboxing class." C. "I cannot be an alcoholic because I still go to work every day." D. "I told my boyfriend if he leaves me, I will kill myself."

A, D Choices A and D are correct. Individuals born before 1957 typically have a natural immunity to the diseases; most older people were exposed to or contracted the diseases. A known allergy to neomycin is a contraindication to the vaccination. Although a contraindication in the past, an egg allergy is no longer considered a contraindication to this vaccine and the treatment seems to be safe for these individuals. Choice B is incorrect. The vaccine contains a live, attenuated virus. Choice C is incorrect. Adults born after 1957 with an unclear immunization history should receive two immunizations one month apart. Even if the individual has received the MMR in the past, there is no danger in receiving the vaccine again.

You are instructing a 65-year-old adult patient about his risk for measles, mumps, or rubella and whether he needs to receive the vaccination. He has a history of allergy to neomycin. Your instruction should include: Select all that apply. A. Due to his age, the patient likely has natural immunity. B. The vaccine does not include a live virus. C. The individual with an unclear immunization history should not receive the vaccine. D. He should not receive the vaccination due to his neomycin allergy.

D Choice D is correct. The state's scope of practice documents for RNs, LPNs, and applicable unlicensed assistive personnel are the legal documents that must be considered when creating assignments or delegating tasks for the day. These legal documents abide by specific state regulations differentiating what healthcare team members may or may not do under the licensure they possess (or, in the case of an UAP, the lack of licensure). Therefore, Choice D is correct. Choice A is incorrect. Although competency checklists for each team member should be considered when making daily assignments and/or delegating tasks, competency checklists are not legal documents. Therefore, Choice A is incorrect. Choice B is incorrect. Although the job descriptions for each team member should be considered when making daily assignments and/or delegating tasks, job descriptions are not legal documents. Therefore, Choice B is incorrect. Choice C is incorrect. Each state specifies the role of RNs and LPN/LVNs via legislation as defined in the Nursing Practice Act. The American Nurses Association does not produce legal documents or create the Nursing Practice Act. Therefore, Choice C is incorrect. Every state and territory in the United States has established laws in place to govern the practice of nursing. These laws are defined in the Nursing Practice Act. The Nursing Practice Act is then interpreted into regulations by each individual state and territorial nursing board with the authority to regulate the practice of nursing care and the power to enforce the laws. Fifty states, the District of Columbia, and four United States territories have state boards of nursing responsible for regulating their individual Nursing Practice Acts. The boards enforce these laws as defined by their respective state or territorial legislative bodies. The legislative body gives the board of nursing the power to discipline nurses who violate the nursing laws and regulations. Only a state board of nursing possesses the power to discipline a nurse whom they believe poses a danger to the public. The American Nurses Association has an established review program for recognition of a nursing specialty, approval of a specialty nursing scope of practice statement, acknowledgment of specialty nursing standards of practice, and affirmation of focused practice competencies.

Today, you are the charge nurse for the nursing care unit overseeing RNs, LPNs, and unlicensed assistive personnel (UAPs). As you prepare the staff assignments prior to the shift, which of the following legal documents must you consider as you are making staff assignments and delegating tasks for the day? A. Competency checklists for all the team members scheduled for the shift B. Job descriptions of all team members on the unit for today's shift C. The scope of practice for both RNs and LPNs as established by the American Nurses Association D. The state's scope of practice documents for RNs, LPNs, and any applicable unlicensed assistive personnel (UAP)

A Choice A is correct. The cylinder must always be checked before use to ensure that enough oxygen is available for the patient. Choice B is incorrect. It is unsafe to use a cylinder that reads 500 psi or less because not enough oxygen remains for a patient transfer. Choice C is incorrect. A cylinder that is not secured correctly may result in injury to the patient during transfer. Choice D is incorrect. Oxygen flow is discontinued by turning the valve clockwise until it is tight.

What action does the nurse perform to follow safe technique when using a portable oxygen cylinder? A. Check the amount of oxygen in the cylinder before using it. B. Use a cylinder for a patient transfer that indicates available oxygen is at 500 psi. C. Place the oxygen cylinder on the stretcher next to the patient. D. Discontinue oxygen flow by turning the cylinder key counter-clockwise until it is tight.

C Choice C is correct. When caring for a patient who is in spiritual distress; the nurse should listen to the patient first. Goals and expected outcomes for patients in spiritual distress need to be individualized and may include a patient achieving some of the following: Exploring the origin of spiritual beliefs and practices Identifying factors in life that challenge spiritual beliefs Exploring alternatives given these challenges: denying, modifying, or reaffirming beliefs, developing new beliefs Identifying spiritual supports Reporting or demonstrating a decrease in spiritual distress after successful intervention Choice A is incorrect. A hug and false reassurances do not address the diagnosis of spiritual distress. Choice B is incorrect. After listening to the patient, the nurse can ask if the patient would like a consultation with a spiritual adviser. However, the nurse should not arrange for a spiritual adviser to visit without the patient's consent. Choice D is incorrect. Talking to friends or relatives may be helpful, but should only be done if the patient expresses wishes to do so.

When a hospice patient tells the nurse, "I feel no real connection with God," what is the nurse's most appropriate response? A. Give the patient a hug and tell her that her life still has meaning B. Arrange for a spiritual adviser to visit the patient C. Ask the patient if she would like to talk about her feelings D. Call in a close friend or relative to talk with the patient

A Choice A is correct. Normal documentation of the assessment of the nose would include findings such as symmetrical, midline, without drainage, and proportional to facial features. Choice B is incorrect. Yellow nasal drainage is never a normal finding. Choices C and D are incorrect. The nose should be symmetrical.

When assessing a client's nose, the normal expected findings should be documented as: A. Nose symmetrical and midline B. Nose symmetrical with yellow drainage C. Nose asymmetrical with clear drainage D. Nose asymmetrical and proportional to facial features

A, B, D, E Choices A, B, D, and E are correct. Amish live a life that is generally strictly separate from society. While women are highly respected and valued, men hold the authority in the home. Traditional and alternative health care is appreciated, although many live without insurance. Health is believed to be a gift from God. Choice C is incorrect. Women are respected in Amish society but do not hold authoritative power.

When caring for an Amish patient, what does the nurse know to be true? Select all that apply. A. They use traditional and alternative health care. B. Funerals are conducted in the home. C. The authority of women and men are equal. D. Many choose to live without health insurance. E. Health is believed to be a gift from God.

C Choice C is correct. The nursing diagnosis that is the most appropriate for an intensive care hospitalized client who is adversely affected with restlessness, anxiety, and feeling the loss of control and who, before this hospitalization, had no psychiatric mental health disorder or these symptoms is "sensory overload related to intensive care hospitalization." The signs and symptoms of sensory overload include restlessness, anxiety, feeling the loss of control, and feeling overwhelmed; clients in intensive care are at risk for sensory overload as a result of their illness and the constant activity and lighting in the critical care or intensive care unit. Choice A is incorrect. This client is not "at risk for sensory deprivation related to acute illness," the client is adversely affected with symptoms that indicate an actual, rather than a risk-related, nursing diagnosis. Choice B is incorrect. This client is not "at risk for sensory overload related to hospitalization," the client is adversely affected with symptoms that indicate an actual, rather than a risk-related, nursing diagnosis. Choice D is incorrect. The signs and symptoms of sensory deprivation include anxiety, depression, hallucinations, and other sensory and perceptual alterations. The signs and symptoms of sensory overload include restlessness, anxiety, feeling the loss of control, and feeling overwhelmed.

Which nursing diagnosis is the most appropriate for an intensive care hospitalized client who is adversely affected with restlessness, anxiety, and feeling the loss of control and who, before this hospitalization, had no psychiatric mental health disorder or these symptoms? A. At risk for sensory deprivation related to acute illness. B. At risk for sensory overload related to hospitalization. C. Sensory overload related to intensive care hospitalization. D. Sensory deprivation related to hospitalization

A, C Choice A is correct. The kidneys cannot excrete excess hydrogen ions or reabsorb bicarbonate with ATN. Due to the inability to excrete the excess acid (hydrogen ions) paired with the inability to hang on to the needed base (bicarbonate), acidosis ensues. This is due to the malfunction of the kidneys, not the lungs, so it is classified as metabolic acidosis. Choice C is correct. ATN can cause hyponatremia. Due to lower urinary output, there is hypervolemia. With fluid retention and high volume remaining in the blood vessels, the amount of sodium in the body is diluted. This is called relative dilutional hyponatremia. Choice B is incorrect. ATN is associated with low thyroxine levels, not high. Thyroid hormones increase renal blood flow and glomerular filtration rate (GFR). In ATN, there is often lower renal blood flow and a lower GR. Therefore, ATN is often associated with low thyroid levels. Choice D is incorrect. ATN can cause increased parathyroid levels. This is considered a secondary hyperparathyroidism. Secondary hyperparathyroidism occurs when the parathyroid glands release too much parathyroid hormone (PTH), causing a high blood level of PTH. This occurs in ATN because when the kidneys are damaged, they cannot make active vitamin D. Vitamin D is required for the absorption of calcium, and calcium levels are therefore low in patients with ATN. One of the primary functions of PTH is the release of calcium from the bones, into the bloodstream, when blood calcium levels are low. The body recognizes the lower blood calcium level, that has been caused by the ATN, and then secrects more PTH to try to correct the issue. This is when ATN can cause increase PTH levels.

Which of the following are complications of acute tubular necrosis (ATN)? Select all that apply. A. Metabolic acidosis B. High thyroxine levels C. Hyponatremia D. Decreased parathyroid levels

C Choice C is correct. The cephalocaudal principle (also known as cephalocaudal development) refers to a general pattern of growth and development followed from infancy into toddlerhood and even early childhood whereby development follows a head-to-toe progression. Choice A is incorrect. Growth occurring from the distal to proximal portions of the body is not a recognized manner or pattern of physical growth or development for any age group, including those ranging from infancy into early childhood. Choice B is incorrect. Growth occurring from the proximal to the distal portions of the body is not a recognized manner or pattern of physical growth or development for any age group, including those ranging from infancy into early childhood. Choice D is incorrect. In children ranging in age from infancy to early childhood, growth does not initially occur most rapidly in the child's extremities. The general pattern of development occurs from the head downward through the body. The cephalocaudal principle applies to both physical and functional development. According to this principle, a child gains control of the head first, then the arms, and then the legs.

Which of the following correctly describes the physical growth pattern from infancy into early childhood? A. Growth occurs from the distal to the proximal parts of the body B. Growth occurs from the proximal to the distal parts of the body C. Growth occurs in a head-to-toe progression D. Growth initially occurs most rapidly in the extremities

D Choice D is correct. Hyperglycemia is a physiological alteration that can occur during a stress response among both diabetic and non-diabetic clients. More specifically, glucose is increased by various factors, including elevated levels of cortisol, glucagon, and epinephrine (often referred to the "fight or flight" phenomena). These hormones may, in turn, lead to insulin resistance, further increasing hyperglycemia. Choice A is incorrect. Increased, rather than decreased, vision is a physiological alteration which occurs during the "fight or flight" phenomena, allowing the individual to be more vigilant and aware of the surrounding environment. This occurs due to the release of adrenaline which causes the pupils to dilate, allowing in more light and make it easier to detect potential threats. Choice B is incorrect. During the body's stress-related response, peristalsis slows (or even stops) to allow the body to divert some (or all) resources to bodily functions deemed higher priority than gastric functioning. Therefore, decreased, not increased peristalsis occurs. Choice C is incorrect. The secretion of glucocorticoids is a classic endocrine response to stress. During the body's stress-related response, an increased, not decreased, secretion of glucocorticoids occurs. Stress-induced hyperglycemia generally refers to transient hyperglycemia during illness and is usually restricted to clients without previous evidence of diabetes. Clients experiencing stress-induced hyperglycemia during hospitalization have been shown to exhibit worse overall outcomes.

Which of the following is a physiological alteration that can occur with stress? A. Decreased visual acuity B. Increased peristalsis C. Decreased glucocorticoids D. Hyperglycemia

C, D, E Choices C, D, and E are correct. Nasal flaring is a sign of respiratory distress. If the newborn is working hard to breathe, they use extra effort to pull air in through their nose, and their nares flare out with inhalation. This signifies they are struggling to breathe and indicates respiratory distress. Head bobbing is a severe sign of respiratory distress in newborns. As they work harder and harder to breathe, they start using the muscles in their neck to pull their head forward with each inhalation. This signifies they are struggling to breathe and indicates respiratory distress. Finally, grunting is a sign of respiratory distress and may be coupled with the infant developing pallor that may transition to cyanosis. Choices A and B are incorrect. Most newborns' primary way to breathe is through their noses. While some evidence may refute this claim, newborns require a patent nasal airway to have an effective gas exchange. Sneezing is a normal finding and does not signify respiratory distress. ✓ The normal respiratory rate for a newborn infant is 30-60 breaths/min Manifestations of respiratory distress include: ✓ Nasal flaring ✓ Grunting ✓ Tachypnea ✓ Central cyanosis ✓ Retractions

Which of the following signs are indicative of respiratory distress in the newborn? Select all that apply. A. Nose breathing B. Occasional sneezing C. Nasal flaring D. Head bobbing E. Grunting

D Choice D is correct. NPH insulin and regular insulin can and are often mixed in the same syringe without the risk of incompatibility. Choice A is incorrect. Dexamethasone and midazolam cannot be mixed in the same syringe because they are not compatible. However, dexamethasone and other medications such as metoclopramide are compatible and can be mixed in the same syringe. Choice B is incorrect. Haloperidol and ketorolac cannot be mixed in the same syringe because they are not compatible. However, haloperidol and other medications such as hydromorphone can be mixed in the same syringe. Choice C is incorrect. Hydrocortisone and midazolam cannot be mixed in the same syringe because they are not compatible. However, hydrocortisone and other medications such as metoclopramide are compatible and can be mixed in the same syringe.

Which of these medications can be mixed in the same syringe without the risk of any incompatibility? A. Dexamethasone and midazolam B. Haloperidol and ketorolac C. Hydrocortisone and midazolam D. NPH and regular insulin

B, C, E Choices B, C, and E are correct. Surgical asepsis is used when managing central line intravenous medication administration, when wearing sterile gloves in the operating room, and when inserting an indwelling Foley catheter. Asepsis is the freedom from disease-causing microorganisms. To decrease the possibility of transferring microbes from one place to another, an aseptic technique is used. The two basic types of asepsis are medical and surgical. Medical asepsis includes all practices intended to confine a specific microorganism to a particular area, limiting the number, growth, and transmission of microorganisms. In medical asepsis, objects are referred to as clean, which means the absence of almost all organisms; or dirty (soiled, contaminated), some of which may be capable of causing infection. Surgical asepsis, or sterile technique, refers to those practices that keep an area or object free of all microorganisms; it includes practices that destroy all microorganisms and spores (microscopic dormant structures formed by some pathogens that are very hardy and often survive common cleaning techniques). Surgical asepsis is used for all procedures involving the sterile areas of the body. Sepsis is the condition in which acute organ dysfunction occurs secondary to infection. Choices A, D, and F are incorrect. Medical asepsis, or clean technique, is used to administer an intramuscular injection, bathing a neonate, and emptying a urinary drainage bag.

Which procedures necessitate the use of surgical asepsis techniques? Select all that apply. A. Intramuscular medication administration B. Central line intravenous medication administration C. Wearing gloves in the operating room D. Neonatal bathing E. Foley catheter insertion F. Emptying a urinary drainage bag

D Choice D is correct. The second method of birth control is necessary until the sperm count is zero. A vasectomy is a form of male birth control that cuts the supply of sperm to your semen. It's done by cutting and sealing the tubes that carry sperm. Vasectomy has a low risk of problems and can usually be performed in an outpatient setting under local anesthesia. Although vasectomy reversals are possible, vasectomy should be considered a permanent form of male birth control. Vasectomy offers no protection from sexually transmitted infections. Vasectomy is a safe and effective birth control choice for men who are sure they don't want to father a child in the future. Vasectomy is nearly 100 percent effective in preventing pregnancy. Vasectomy is an outpatient surgery with a low risk of complications or side effects. The cost of a vasectomy is far less than the price of female sterilization (tubal ligation) or the long-term value of birth control medications for women. Choice A is incorrect. Although reversal is possible, it is often difficult, requiring microsurgery. Also, results may be unsuccessful. Choice B is incorrect. Once the sperm count is zero, there is no need for follow-up exams. Choice C is incorrect. There is no correlation between having a vasectomy and cardiac disease.

Which statement should the nurse use during client education regarding a vasectomy as a permanent method of contraception? A. If you change your mind in the future, it's simple to reverse the procedure. B. You will need to return for an annual follow-up visit and sperm count. C. If you have a history of cardiac disease. we won't be able to do the vasectomy. D. You'll need to use another type of birth control until your sperm count is zero.

A, C Choices A and C are correct. A fetal heart rate less than 110 beats/minute or greater than 160 beats/minute is nonreassuring (Choice A). Late decelerations are an ominous sign, therefore, immediate interventions should be taken to improve the fetal heart rate; they are characteristic of a nonreassuring heart rate (Choice C). Choice B is incorrect. An increase in variability is a reassuring factor. A decrease in variability would be nonreassuring. Choice D is incorrect. Mild, variable decelerations are okay, only when the variable decelerations are severe are they nonreassuring.

While reviewing fetal monitoring strips, the labor and delivery nurse notes that the reading is nonreassuring. What features characterize a fetal monitoring strip as nonreassuring? Select all that apply. A. Fetal heart rate less than 110 beats/minute. B. Increase in variability. C. Late decelerations D. Mild variable decelerations

Place the infant in the knees to chest position Administer 100% oxygen Administer morphine sulfate Administer an IV fluid bolus Document the event Correct answer: The priority in a hypercyanotic tet spell is to place the child in a knee to chest position. Tet spells occur when the infant with tetralogy of Fallot becomes acutely cyanotic due to infundibular spasm usually associated with feeding or crying. When this spasm occurs, there is decreased flow from the right ventricle due to the obstruction, resulting in severe hypoxia. Putting the child in a knee-chest position increases the intrathoracic pressure and increases blood flow to the lungs, therefore increasing oxygenation to body tissues. The next priority action is to administer 100% oxygen to assist in meeting the child's oxygenation requirements and relieving the hypoxia quickly. The following priority action is to administer morphine sulfate. This is the drug of choice for tet spells because it helps to calm the child down while simultaneously reducing the infundibular spasm that causes right ventricular outflow obstruction and, therefore, the hypercyanotic tet spell. The next priority nursing action is to administer an IV fluid bolus. This increases preload and consequently, cardiac output, helping to increase perfusion and oxygenation to the tissues. Lastly, the nurse should document the event, actions taken, and the patient's response.

While working in a pediatric cardiac intensive care unit, you are caring for a child diagnosed with tetralogy of Fallot. Upon entering the room in the morning for your first assessment you find the child crying, cyanotic, and tachycardic. You recognize this as a hypercyanotic tet spell. Place the following actions in order of priority: -Administer 100% oxygen -Place the infant in the knees to chest position -Administer an IV fluid bolus -Administer morphine sulfate -Document the event

A, B, C Choices A, B, and C are correct. A is correct. This is an excessive amount of urine output for 1 hour and is concerning for diabetes insipidus given the procedure the patient recently underwent. Any urine output greater than 300 mL is alarming and the healthcare provider should be notified immediately. Diabetes insipidus is a severe complication from neurosurgery that occurs around the pituitary. This amount of urinary output can lead to shock if not treated promptly. B is correct. Requesting an order for IV fluids is an appropriate nursing action given your assessment. You are concerned about the possibility of DI considering the excessive urine output and there is no fluid replacement currently ordered for this patient. This is concerning for shock, and IVF should be initiated to rehydrate and adequately replace losses from the urinary output. C is correct. These findings should be accurately documented to ensure proper follow-up and orders for this patient. Choice D is incorrect. No oxygen therapy is indicated for this patient at this time. His O2 saturation is adequate on room air, he is not tachypneic, and the question stem gave you no other information to indicate that there was an increased work of breathing or oxygen requirement.

While working in the PICU, you are assigned to a 12-year-old male who is 1-hour post-op from transsphenoidal hypophysectomy. He has 2 PIV's, is on room air, has an NG tube, and a foley catheter. You complete your assessment and note the following: HR: 141 RR: 24 Temp: 37.1 O2: 99% PIV's: Patent and saline locked NG: No drainage, clamped Foley: 400 mL of clear urine Which of the following actions are appropriate given your assessment? Select all that apply. A. Notify the health care provider of the urine output. B. Request an order for IV fluids. C. Document your findings. D. Initiate NC 2L O2 at 100%.

B Choice B is correct. You should teach the husband about Alzheimer's and the need to promote as much independence as possible. Adults diagnosed with dementia are faced with a disease that is irreversible and progressive. The loss of judgment, reasoning, memory, and communication skills leads to an inability to discern risk and danger. Dementia can limit a person's ability to live independently, which can be very distressing for the individual and family members. Caregivers need to embrace a patient-centered approach that allows people with dementia to maintain as much autonomy and control as possible, while still preserving their safety. Choice A is incorrect. Moving closer to the children may not be appropriate advice, mainly if the children are unable or unwilling to care for their mother. Choice C is incorrect. Client's with Alzheimer's disease and other disabilities, including physical disabilities, should be coached and encouraged to be as independent as possible. Choice D is incorrect. The couple should be advised to continue their social activities.

You are a home health nurse caring for an elderly client in her home. She has children and grandchildren. However, they live far from the couple and they typically visit only once or twice a year. The client is beginning to show some signs of Alzheimer's. The husband is 88-years-old and had a stroke that left him with right-sided weakness. What support should you give the husband in terms of caring for his wife? A. You should advise the couple to move closer to their children so that they can care for their father. B. You should teach the husband about the progression of Alzheimer's and the need to promote as much independence as possible. C. You should teach the husband about this progressive disease and the need to do all that he can for his wife to help prevent anxiety and depression. D. You should advise the couple to decrease their social activities in order to preserve the wife's dignity and self-esteem.

B Choice B is correct.This ABG shows a respiratory acidosis. The first clue in this patient is the diagnosis of COPD. In COPD, the patient suffers from severe hypoventilation. This hypoventilation results in the retention of carbon dioxide. The registered nurse must know the basics of ABG interpretation, including the normal ranges for each of the values. First, the nurse should look at the pH. The normal range is 7.35-7.45. A value below 7.35 indicates an acidosis; a value above 7.45 indicates an alkalosis. The normal partial pressure of carbon dioxide (PaCO2) is 35-45 mmHg. Standard bicarbonate for a man this age is 22-29 mmol/L. The pH in this patient shows that the condition is acidosis. The high PaCO2 indicates that it is a respiratory problem. These values would support the assumption based on the diagnosis of COPD. The pH and PaCO2 define respiratory disorders. Respiratory acidosis is defined as a pH below 7.35 and a PaCO2 above 45 mmHg. Respiratory alkalosis is defined as a pH above 7.45 and a PaCO2 below 35 mmHg. Metabolic disorders are defined by the pH and the bicarbonate (HCO3). Metabolic acidosis is defined as a pH below 7.35 and an HCO3 below 22 mmol/L. Metabolic alkalosis is defined as a pH above 7.45 and an HCO3 above 29 mmol/L.

You are caring for a 55-year-old male patient in the emergency department. He has a history of chronic obstructive pulmonary disease (COPD). He came to the ED with a complaint of shortness of breath. His respiratory rate is 28 per minute, and his breaths are shallow and somewhat difficult. You put him on supplemental oxygen at 2 L/minute. You draw ABGs. You receive results of the arterial blood gas that show: pH = 7.30 PaCO2 = 49 Bicarbonate = 25 You determine that this ABG shows: A. Metabolic alkalosis B. Respiratory acidosis C. Respiratory alkalosis D. Metabolic acidosis

A, D Choices A and D are correct. These statements indicate a need for further education. During an asthma attack, the first action should not be to call 911. The patient will have an asthma action plan that lists the steps she should take in the order she should take them. For most patients, the first step is to take short-acting inhaler medications. It is not necessary to first call 911 for every asthma attack (Choice A). Although asthma will not stop every child from getting a dog, pets with hair that sheds can be a trigger. It would be inadvisable for a teen newly diagnosed with asthma to get a new dog. It could end up causing more asthma attacks and present a severe problem. If the patient wants a new pet, a fish would be a better recommendation given their new asthma diagnosis (Choice D). Choice B is incorrect. This is an appropriate statement and does not indicate a need for further education. When a patient is having an asthma attack, the physiology includes inflammation and constriction of the airways. This can result in obstruction, making it impossible for the patient to breathe. That is why asthma attacks are so dangerous. Choice C is incorrect. This is an appropriate statement and does not indicate a need for further education. One of the most critical educational points for patients newly diagnosed with asthma is identifying their triggers. Triggers are what precipitate an asthma attack for that patient. For example, maybe playing soccer, or dusting the house. Whatever it is that precipitates their asthma should be avoided.

You are providing asthma education to a teen that has just been diagnosed with asthma. Which of the following statements indicate a need for further teaching? Select all that apply. A. "When I am having an asthma attack, I should call 911 first." B. "When I am having an asthma attack, my airway is constricting and it can become dangerous." C. "I should try to identify what causes me to have an asthma attack and avoid those activities." D. "I've really been wanting to get a dog and my asthma will not stop me."

C Choice C is correct. You would call the doctor to report this urinary oliguria and initiate hourly urinary output measurements because 150 mL over 8 hours is less than 19 mL per hour and less than 450 mL for 24 hours. This output is considered oliguria because the expected urinary production for an adult client is about 1,500 mL per day. Additionally, a urinary output of less than 19 mL per hour is a significant finding that can indicate a severe medical problem; therefore, the doctor must be notified immediately. Choice A is incorrect. You would not merely record the urinary output according to your facility's policy and procedure; there is something else that you must do in addition to this recording and documentation. Choice B is incorrect. You would not merely report this urinary output to the oncoming shift as part of your "hand-off" report; there is something else that you must do in addition to this reporting. Choice D is incorrect. You would not call the doctor and report this urinary output as part of your daily doctor's update; there is another reason why you would call the doctor.

You are serving as the preceptor for a new graduate nurse. This new nurse is caring for a small group of adult clients under your supervision. Your shift is 8 hours long. The intake and output of clients are calculated and documented at the end of the shift. The new nurse reports a total urinary production of 150 mL from the urinary drainage bag for your 58-year-old male postoperative client at the end of your shift. What should you do? A. Simply record the urinary output according to your facility's policy and procedure. B. Simply report this urinary output to the oncoming shift as part of your "hand-off" report. C. Call the doctor to report this urinary oliguria and initiate hourly urinary output measurements. D. Call the doctor and report this urinary output as part of your daily doctor's update.

Call for help and stay with the patient. Cover the wound with a sterile normal saline dressing. Take vital signs and monitor for signs of shock. Prepare the patient for immediate surgery. Document the incident. The priority of nursing action is to call for help but stay with the patient. The nurse should tell the person who responds to notify the surgeon immediately. This is a surgical emergency, therefore the surgeon must be notified STAT. After help has been called, the nurse needs to cover the wound with a sterile 0.9% sodium chloride dressing. This helps prevent infection and keep the protruding organ moist and hydrated before surgery. The nurse should instruct the patient not to strain or cough, and keep the client in low Fowler's position (no more than 20 degrees of bed elevation) with his/her knees flexed. This position relaxes abdominal muscles and reduces abdominal muscle tension. After this, the next nursing action is to check the patient's vital signs and monitor for shock while waiting for the health care providers. If signs of shock such as tachycardia and hypotension are noted, this is a medical emergency, and the health care provider/rapid response team needs to be called to the bedside immediately. After taking vital signs, the nurse should begin preparing the patient for immediate surgery. Lastly, after the patient has been taken to surgery, the nurse needs to document the incident.

You are taking care of an 80-year-old patient who is post-op day one from abdominal surgery. Upon assessment, you notice bowel protruding through her incision and quickly determine that evisceration has occurred. Place the following actions in order of priority: Call for help and stay with the patient. Cover the wound with a sterile normal saline dressing. Prepare the patient for immediate surgery. Take vital signs and monitor for signs of shock. Document the incident.

B, D Choices B and D are correct. This is an appropriate teaching point. Regular insulin is the standard insulin given IV. It is active for about 6 to 8 hours and peaks in 2-4 hours. Regular insulin is clear. NPH insulin is considered intermediate-acting insulin. It is active for about 16-18 hours and peaks in 4-10 hours. NPH insulin is cloudy. It is safe to administer regular insulin and NPH insulin together in the same syringe, but they must be drawn up correctly. You should teach the nurses first to draw up regular insulin, and then draw up the NPH insulin (Choice B). Monitoring of the HbA1c is crucial in diabetic patients and you should educate your patient about the need to check this level at their doctor's appointments. Glycosylated Hemoglobin (HbA1c) shows the percentage of red blood cells that have become saturated with hemoglobin. The higher this number is, the higher the patient's blood sugar has been over the past 3-4 months. Anything higher than 6.5% indicates that the patient has diabetes. For patients with diagnosed diabetes, their goal should a HbA1c of less than 7% and they should have it checked every 3-4 months (Choice D). Choice A is incorrect. Glargine, or Lantus, is a long-acting insulin; its duration of action is 24 hours and does not have a peak. Due to this, it is given just once a day and acts as the basal insulin for the client. Glargine can be provided in the morning or evening, as long as the client takes it at the same time every day. Since there is no peak, glargine does not need to be timed with meals, so you would not teach the new nurses to wait for the client's food before administration. Choice C is incorrect. You should teach patients to eat 2-4 hours after taking their regular insulin. The peak of regular insulin is 2-4 hours, so we should teach patients to eat when insulin is at its peak level. When insulin is at its peak, blood sugar is at its lowest. This is the most appropriate timing for regular insulin administration and meals.

You are teaching a new group of nurses about insulin administration for a client with type I diabetes mellitus. Which of the following points should you include? Select all that apply. A. It is important to wait for the client's food tray to be delivered before administering their glargine. B. When drawing up different types of insulin together in a syringe, first draw up regular insulin and then NPH. C. You should teach patients to eat 1-2 hours after taking their regular insulin. D. Monitoring the HbA1c is very important and patients with diabetes should have a goal of less than 7%.

A Choice A is correct. You should integrate the knowledge that the aging population is more at risk for an accidental overdose to medications when compared to other age groups. This risk for an unintentional overdose of drugs occurs due to some of the regular changes in the aging process, such as decreased metabolism. Choice B is incorrect. You would not integrate the knowledge that the aging population is more at risk for low therapeutic levels of medications than other age groups because this is not true. Choice C is incorrect. You would not integrate the knowledge that the aging population cannot swallow medications, so all these medications should be crushed and placed in apple sauce or pudding before administration. Some medicines cannot be broken and placed in apple sauce or pudding before administration, and many members of the elderly population can swallow pills and tablets. Choice D is incorrect. You would not integrate the knowledge that the aging population often rejects their medications, so all these medications should be crushed and placed in apple sauce or pudding to conceal them. Although clients have the right to refuse drugs, this is rare; additionally, concealing medications is unethical.

You are working with geriatric clients in a long-term care facility. What knowledge should you continuously integrate into your role as the nurse administering medications to the aging population? A. The knowledge that the elderly population is more at risk for an accidental overdose than other age groups. B. The knowledge that the elderly population is more at risk for low therapeutic levels of medications than other age groups. C. The knowledge that elderly clients cannot swallow medications, so all these medications should be crushed and placed in apple sauce or pudding before administration. D. The knowledge that elderly clients often reject their medications, so all these medications should be crushed and placed in apple sauce or pudding to conceal them.


Conjuntos de estudio relacionados

lecture 5: Selecting a Jury and Jury decision making

View Set

Socia Psychology Quiz 1-3 Questions

View Set

FINC 350 ch 10,11,12 homework problems

View Set